17.07.2024

Когда возникает режим короткого замыкания в цепи: Короткое замыкание – определение и формула для электрического тока кратко (физика 8 класс)

Содержание

Причины возникновения и последствия коротких замыканий

Короткое замыкание возникает при соединении двух проводов цепи, присоединенных к разным зажимах (например, в цепях постоянного тока это «+» и ««) источника через очень малое сопротивление, которое сравнимо с сопротивлением самих проводов.

Ток при коротком замыкании может превысить номинальный ток в цепи во много раз. В таких случаях цепь должна быть разорвана раньше, чем температура проводов достигнет опасных значений.

Для защиты проводов от перегрева и предупреждения воспламенения окружающих предметов в цепь включаются аппараты защиты — плавкие предохранители или автоматические выключатели.

Короткие замыкания могут возникнуть также при перенапряжениях в результате грозовых явлений, прямых ударов молнии, механических повреждении изолирующих частей, ошибочных действий обслуживающего персонала.

При коротких замыканиях резко возрастают токи в короткозамкнутой цепи и снижается напряжение, что представляет большую опасность для электрического оборудования и может вызвать перебои в электроснабжении потребителей.

Короткие замыкания бывают:

  • трехфазные (симметричные), при которых накоротко замыкаются все три фазы

  • двухфазные (несимметричные), при которых накоротко замыкаются только две фазы

  • двухфазные на землю в системах с глухо заземленными нейтралями

  • однофазные несимметричные на землю заземленными нейтралями

Наибольшей величины ток достигает при однофазном коротком замыкании. В результате применения специальных искусственных мер (например заземления нейтралей через реакторы, заземление только части нейтралей) наибольшее значение тока однофазного короткого замыкания может быть снижено до величины тока трехфазного короткого замыкания, для которого чаще всего и ведутся расчеты.

Коротким замыканием называется соединением отдельных фаз между собой или с землей через относительно малое сопротивление, принимаемое равным нулю при глухом металлическом коротком замыкании

Причины возникновения коротких замыканий

Основной причиной возникновения коротких замыканий является нарушения изоляции электрооборудования.

Нарушения изоляции вызываются:

  • Перенапряжениями (особенно в сетях с изолированными нейтралями)
  • Прямыми ударами молнии
  • Старением изоляции
  • Механическими повреждениями изоляции, проездом под линиями негабаритных механизмов
  • Неудовлетворительным уходом за оборудованием

Часто причиной повреждений в электрической части электроустановок являются неквалифицированные действия обслуживающего персонала.

Преднамеренные короткие замыкания

При осуществлении упрощенных схем соединений понижающих подстанций используют специальные аппараты — короткозамыкатели, которые создают преднамеренные короткие замыкания с целью быстрых отключений возникших повреждений. Таким образом, наряду с короткими замыканиями случайного характера в системах электроснабжения имеют место также преднамеренные короткие замыкания, вызываемые действием короткозамыкателей.

Последствия коротких замыканий

В результате возникновения короткого замыкания токоведущие части сильно перегреваются, что может привести к нарушению изоляции, а также возникновению больших механических усилий, способствующих разрушению частей электроустановок.

При этом нарушается нормальное электроснабжение потребителей в неповрежденных участках сети, так как аварийный режим короткого замыкания в одной линии приводит к общему снижению напряжения. В месте короткого замыкания спряжение становится равным нулю, а во всех точках до места короткого замыкания напряжение резко снижается, и нормальное питание неповрежденных линий становится невозможным.

При возникновении коротких замыканий в системе электроснабжения ее общее сопротивление уменьшается, что приводит к увеличению токов в ее ветвях по сравнению с токами нормального режима, а это вызывает снижение напряжения отдельных точек системы электроснабжения, которое особенно велико вблизи места короткого замыкания. Степень снижения напряжения зависит от работы устройств автоматического регулирования напряжения и удаленности от места повреждения.

В зависимости от места возникновения и продолжительности повреждения его последствия могут иметь местный характер или отражаться на всей системе электроснабжения.

При большой удаленности короткого замыкания величина тока короткого замыкания может составлять лишь незначительную часть номинального тока питающих генераторов и возникновение такого короткого замыкания воспринимается ими как небольшое увеличение нагрузки.

Сильное снижение напряжения получается только вблизи места короткого замыкания, в то время как в других точках системы электроснабжения это снижение менее заметно. Следовательно, при рассматриваемых условиях опасные последствия короткого замыкания проявляются лишь в ближайших к месту аварии частях системы электроснабжения.

Ток короткого замыкания, являясь даже малым по сравнению с номинальным током генераторов, обычно во много раз превышает номинальный ток ветви, где произошло короткое замыкание. Поэтому и при кратковременном протекании тока короткого замыкания он может вызвать дополнительный нагрев токоведущих элементов и проводников выше допустимого.

Токи короткого замыкания вызывают между проводниками большие механические усилия, которые особенно велики в начале процесса короткого замыкания, когда ток достигает максимального значения. При недостаточной прочности проводников и их креплений могут иметь место разрушения механического характера.

Внезапное глубокое снижение напряжения при коротком замыкании отражается на работе потребителей. В первую очередь это касается двигателей, так как даже при кратковременном понижении напряжения на 30-40% они могут остановиться (происходит опрокидывание двигателей).

Опрокидывание двигателей тяжело отражается на работе промышленного предприятия, так как для восстановления нормального производственного процесса требуется длительное время и неожиданная остановка двигателей может вызвать брак продукции предприятия.

При малой удаленности и достаточной длительности короткого замыкания возможно выпадение из синхронизма параллельно работающих станций, т.е. нарушение нормальной работы всей электрической системы, что является самым опасным последствием короткого замыкания.

Возникающие при замыканиях на землю неуравновешенные системы токов способны создать магнитные потоки, достаточные для наведения в соседних цепях (линиях связи, трубопроводах) значительных ЭДС, опасных для обслуживающего персонала и аппаратуры этих цепей.

Таким образом, последствия коротких замыканий следующие:

  • Механические и термические повреждения электрооборудования
  • Возгорания в электроустановках
  • Снижение уровня напряжения в электрической сети, ведущее к уменьшению вращающего момента электродвигателей, их торможению, снижению производительности или даже к опрокидыванию их
  • Выпадение из синхронизма отдельных генераторов, электростанций и частей электрической системы и возникновение аварий, включая системные аварии
  • Электромагнитное влияние на линии связи, коммуникации и т.п

Для чего нужен расчет токов короткого замыкания

Короткое замыкание цепи вызывает переходный процесс в ней, в ходе которого ток можно рассматривать как сумму двух составляющих: вынужденной гармонической (периодической, синусоидальной) iп и свободной (апериодической, экспоненциальной) iа. Свободная составляющая уменьшается с постоянной времени Тк = Lк/rк = xк/ωrк по мере затухания переходного процесса. Максимальное мгновенное значение iу суммарного тока i называется ударным током, а отношение последнего к амплитуде Iпm — ударным коэффициентом.

Вычисление токов короткого замыкания необходимо для правильного выбора электрооборудования, проектирования релейной защиты и автоматики, выбора средств ограничения токов короткого замыкания.

Короткие замыкания (КЗ) происходят обычно через переходные сопротивления — электрических дуг, посторонних предметов в месте повреждения, опор и их заземлений, а также сопротивления между проводами фаз и землей (например, при падении проводов на землю). Для упрощения расчетов отдельные переходные сопротивления в зависимости от вида повреждения принимаются равными между собою или равными нулю («металлическое», или «глухое» КЗ).

Короткое замыкание и перегрузка, в чем их различие

Короткое замыкание и перегрузка, в чем их различие

Короткое замыкание (КЗ) — электрическое соединение двух точек электрической цепи с различными значениями потенциала, не предусмотренное конструкцией устройства и нарушающее его нормальную работу.

Если фазу и ноль электрической сети соединить под напряжением друг с другом не через потребитель, а напрямую, то возникнет короткое замыкание, сокращенно КЗ.

Коротким замыканием называется соединение проводников отдельных фаз между собой или с землей через относительно малое сопротивление, принимаемое равным нулю при глухом металлическом коротком замыкании.

Никакая сеть не предназначена для длительной работы в таком режиме. Однако данный аварийный режим иногда возникает. Так, короткое замыкание может случиться из-за нарушения изоляции электропроводки или из-за случайного замыкания разноименных проводников проводящими частями электрооборудования. Нормальная работа электрической сети будет нарушена. Чтобы это нежелательное явление предотвратить, электрики используют клеммники либо просто изолируют соединения.

Проблема режима КЗ заключается в том, что в момент его возникновения в сети многократно увеличивается ток (до 20 раз превышает номинал), что приводит к выделению огромного количества джоулева тепла (до 400 раз превышает норму), поскольку количество выделяемой теплоты пропорционально квадрату тока и сопротивлению потребителя.

Теперь представьте: сопротивление потребителя здесь — доли ома проводки, а ток, как известно, тем выше, чем меньше сопротивление. В итоге, если мгновенно не сработает защитное устройство, произойдет чрезмерный перегрев проводки, провода расплавятся, изоляция воспламенится, и может случиться пожар в помещении. В соседних помещениях, питаемых этой же сетью, упадет напряжение, и некоторые электроприборы могут выйти из строя.

Типичный вид короткого замыкания для жилых квартир — однофазное короткое замыкание, когда фаза смыкается с нулем. Для сетей трехфазных, например в цеху или в гараже, возможно трехфазное или двухфазное короткое замыкание (две фазы между собой, три фазы между собой, или несколько фаз на ноль).

Для трехфазного оборудования, такого как асинхронный двигатель или трехфазный трансформатор, характерно межвитковое замыкание, когда витки замыкаются накоротко внутри обмотки статора или внутри обмотки трансформатора, шунтируя остальные рабочие витки и выводя таким образом прибор из строя.

Или замыкание может случиться через проводящий корпус прибора. Вообще проводящие корпуса следует заземлять, дабы защитить персонал от случайного поражения током, а провода в квартирах использовать те, что в негорючей изоляции.

Есть еще один вид аварийного режима нагрузки электрической сети, связанный с превышением нормального тока. Это так называемая перегрузка. Перегрузки иногда возникают в квартирах, в домах, на предприятиях. Это опасный режим, порой более опасный, чем короткое замыкание. Ведь короткое замыкание в квартире может быть на корню остановлено мгновенно сработавшим автоматическим выключателем в щитке. А вот токовая перегрузка — случай более хитрый.

Представьте себе, что в одну единственную розетку вы решили понавтыкать множество электроприборов через тройник да через удлинители. Что нежелательного может в этом случае произойти? Если жила проводки, подведенный к розетке, не рассчитана на ток более 16 ампер, то при включении в такую розетку нагрузки более 3500 ватт начнется перегрев электропроводки чреватый пожаром.

Вообще тепловое воздействие на изоляцию проводов резко снижает ее механические и диэлектрические свойства. Например, если проводимость электрокартона (как изоляционного материала) при 20°С принять за единицу, то при температурах 30, 40 и 50°С она увеличится в 4, 13 и 37 раз соответственно.

И тепловое старение изоляции наиболее часто возникает именно из-за перегрузки электросетей токами, превышающими длительно допустимые для данного вида и сечений проводников. Также нельзя в розетку, на которой указано 250 В 10 А, включать потребителей более чем на 2500 Вт, ибо может начаться перегрев контактов, ведущий к их ускоренному окислению.

Для защиты от перегрузок в квартире, а также для мгновенного купирования режима КЗ, используйте автоматические выключатели. 

Ранее ЭлектроВести писали, что ДТЭК Киевские электросети возобновил подачу электроэнергии потребителям Печерского района в Киеве. Благодаря оперативным действиям специалистов компании электроснабжение возобновлено на более 20 улицах центральной части Киева.

По материалам: electrik.info.

Что называется коротким замыканием. Причины возникновения короткого замыкания

КЗ образуется вследствие замыкания двух проводов цепи, которые подсоединены к разным контактам (это плюс и минус). В данном случае происходит это через маленькое сопротивление, которое можно сравнить с сопротивлением самого провода. При этом ток может превысить номинальное значение в несколько раз. Чтобы предотвратить возгорание, электрическая цепь должна быть разорвана до того, как провода нагреются до критической температуры.


Что такое короткое замыкание?

Ежедневно, где бы мы не находились, мы осуществляем замыкание электрической цепи. При этом ничего опасного не происходит, так как при подсоединении вилки электрооборудования в розетку электрическая энергия превращается в:

  • механическую энергию;
  • тепловую мощность.

Данные виды замыкания можно условно назвать «длинными». Короткое замыкание — это, говоря простым языком, такой вид энергии, которая выражается в виде искры, хлопка или возгорания. Это такое состояние, когда сопротивление самой нагрузки становится меньше сопротивления источника питания. При коротком замыкании мгновенно увеличивается сила тока, которая приводит к сильному выделению тепла. Это — в свою очередь — может привести к расплавлению проводки и её последующему возгоранию. Такое КЗ способно не только нарушить работоспособность элемента электрической цепи, но и привести к снижению входного напряжения у других потребителей.

В нормальном рабочем режиме ток между фазным и нулевым проводом протекает лишь в том случае, когда подсоединена нагрузка, которая и осуществляет его ограничение на безопасном уровне для электрической проводки. Как происходит короткое замыкание? В тех случаях, когда появляется нарушение изоляционного покрытия, приводящее к замыканию плюса и минуса, ток минует нагрузку и течёт между этими проводами. Данный вид контакта называется «коротким», в связи с тем, что минует электрические приборы.

Металлическое короткое замыкание — это такое замыкание, в котором не учитывается переходное сопротивление. Оно возможно только в случае его специальной подготовки при помощи болтового соединения токоведущих частей.

Ток короткого замыкания — это такой ток, который появляется вследствие повреждения изоляции токоведущих частей, обладающих различным электрическим потенциалом. Возникнуть он может и просто при случайном соединении проводящих частей с теми же потенциалами.

Ударный ток короткого замыкания — это максимальная величина тока, которая возникает при трёхфазном КЗ.

Режим короткого замыкания — это такое состояние двухполюсника, когда его выходы соединены между собой при помощи проводника с нулевым сопротивлением. В данном режиме вторичная обмотка замыкается накоротко. При проведении такого опыта можно определить величину потерь в обмотках самого трансформатора.

Также стоит знать, что напряжение короткого замыкания трансформатора — это такое напряжение, которое необходимо подать на обмотку, когда вторая замкнута. И тогда в последней обмотке начнёт протекать номинальный ток.

Как его обнаружить и предотвратить?

Можно вспомнить всем известный закон Ома, который гласит: «Ток в цепи прямо пропорционален напряжению и обратно пропорционален сопротивлению». Как раз на последнее и стоит обращать в данном случае пристальное внимание. В связи с тем, что сопротивление проводки очень мало, его принято считать равным «0». В случае с КЗ его величина — наоборот — очень велика, так как в замкнутой цепи начинает течь ток.

Для того чтобы предотвратить короткое замыкание, необходимо периодически производить замеры сопротивления проводки. Если вы самостоятельно не можете это делать, то стоит обратиться за помощью к специалистам. Они на профессиональном уровне проведут все измерения, касающиеся проводки, а также помогут провести испытание измерительных трансформаторов тока, что также убережет ваше оборудование и повысит пожарную безопасность.

Рассмотрим особый случай параллельного соединения проводников — так называемое короткое замыкание.
Им называется параллельное включение в цепь проводника с очень маленьким сопротивлением. Рассмотрим пример.
Пусть лампы и выключатель соединены так, как показано на схемах. Обратите внимание, что выключатель и вторая лампа соединены параллельно, кроме того, замкнутый выключатель на правой схеме — проводник с очень маленьким сопротивлением. Следовательно, согласно определению, на правой схеме существует короткое замыкание лампы.

Пусть, например, напряжение источника тока подобрано так, что при разомкнутом выключателе обе лампы светятся не очень ярко — в полнакала (поэтому на первой схеме они наполовину закрашены). Если же выключатель замкнуть, то левая лампа будет гореть ярко, а правая лампа вообще погаснет. Таким образом, увеличение яркости левой лампы указывает нам, что при существовании в цепи короткого замыкания сила тока резко возрастает.
Согласно закону Джоуля-Ленца, возрастание силы тока может привести к перегреванию проводов и возникновению пожара.
Объясним, почему левая лампа загорается ярче. Вспомним, что при параллельном соединении проводников их общее сопротивление становится меньше меньшего из них, то есть даже меньше, чем сопротивление выключателя (у которого оно и так почти равно нулю). Согласно закону Ома, уменьшение сопротивления приводит к возрастанию силы тока. А возрастание тока, согласно закону Джоуля-Ленца, приводит к более сильному накалу спирали левой лампы.
Объясним теперь, почему гаснет правая лампа. Поскольку при параллельном соединении проводников напряжение на каждом из них одинаково, то напряжения на правой лампе и на выключателе одинаковы. По закону Ома U=I·R. Как мы выяснили в предыдущем абзаце, сопротивление этого соединения почти равно нулю, то есть R»0. Подставляя ноль в формулу, получим: U=I·0=0. То есть, напряжение на выключателе и лампе равно нулю (точнее, очень маленькое). Такого напряжения явно недостаточно, чтобы поддерживать свечение лампы, поэтому она гаснет.

Для защиты электроприборов от короткого замыкания применяют предохранители.
Их назначение — отключать электроэнергию в случае, если ток возрастает больше допустимой величины. На рисунке справа вы видите автоматический
предохранитель с винтовым цоколем как у лампы. Такие предохранители (в просторечии «пробки») вворачивают в специальные патроны, которые укрепляют на стене.
Существуют также плавкие предохранители.
В них основной деталью является тонкая (диаметром около 0,1 мм) проволочка из олова или свинца (см. рисунок ниже). В случае сильного возрастания тока она практически мгновенно плавится, и цепь размыкается, прерывая ток. В отличие от «многоразовых» автоматических предохранителей, плавкие предохранители являются одноразовыми электроприборами.

Если предположить, что провода, подводящие ток к квартирной проводке, сделаны из алюминия и имеют диаметр 1 мм, то площадь сечения свинцовой проволочки окажется в 100 раз меньше. Кроме того, заглянув в таблицу, мы увидим, что удельное сопротивление свинца примерно в 10 раз больше, чем у алюминия. Следовательно, сопротивление проволочки примерно в 1000 раз больше сопротивления алюминиевого провода такой же длины.
Поскольку провод и предохранитель (то есть проволочка внутри него) соединены последовательно, то сила тока в них одинакова. Так как по закону Джоуля-Ленца Q=I2Rt, следовательно, количество теплоты, выделяющееся в проволочке, в каждый момент времени в 1000 раз больше, чем в проводе. Именно поэтому проволочка плавится, а электропроводка остаётся в сохранности. В настоящее время плавкие предохранители практически не применяются в технике, уступив место автоматическим.

Любой человек, чья работа связана с обслуживанием электротехники, очень хорошо знает о тех неприятностях, которые таит в себе короткое замыкание (к.з.). Иногда считается, что оно представляет собой повреждение. Это не так. Короткое замыкание — это процесс, или, если угодно, аварийный режим работы какого-либо участка электроустановки. А вот последствия его действительно приводят к повреждениям. Общепринятое определение гласит: «Короткое замыкание — это непосредственное соединение двух или более точек электрической цепи, обладающих различным потенциалом. Является ненормальным (непредусмотренным) режимом работы».

Чтобы понять, что именно происходит в цепи в тот момент, когда там возникает короткое замыкание, необходимо вспомнить принципы функционирования элементов контура. Представим простейшую цепь, состоящую из двух проводников и нагрузки (например, лампочки). В обычных условиях в проводнике существует направленное движение заряженных элементарных частиц, обусловленное постоянным воздействием источника. Они перемещаются от одного полюса источника к другому через два участка провода и лампу. Соответственно, лампа излучает свет, так как частицы совершают в ней определенную работу.

При направление движения постоянно изменяется, но в данном случае это не принципиально. Количество электронов, проходящих по определенному участку цепи за единицу времени, ограничивается сопротивлением лампы, проводников, источника ЭДС. Другими словами, ток не растет бесконечно, а соответствует установившемуся режиму.

Но вот по какой-либо причине повреждается изоляция на участке цепи. К примеру, лампу залило водой. В этом случае ее уменьшается. В результате текущий по контуру ток ограничивается суммарным сопротивлением источника питания, проводов и водного «перешейка» на лампе. Обычно эта сумма настолько ничтожна, что в расчетах не учитывается (исключение составляют специализированные вычисления).

Итогом является практически бесконечный рост тока, определяемого по классическому закону Ома. В данном случае часто упоминают мощность короткого замыкания. Она определяется предельным значением электрического тока, который способен выдать источник питания до выхода из строя. Кстати, именно поэтому запрещается соединять проводком (закорачивать) противоположные контакты батареек.

Хотя в примере мы рассматриваем устранение из цепи сопротивления лампы вследствие попадания на нее воды, причин короткого замыкания множество. К примеру, если говорить об этой же схеме, то к.з. также может возникнуть, если будет нарушена изоляция хотя бы одного провода и он соприкоснется с землей. В этом случае ток от источника питания последует по пути наименьшего сопротивления, то есть в землю, обладающую огромной емкостью. Повреждение изоляции сразу двух проводов и их соприкосновение приведет к тому же самому результату.

Вышесказанное можно обобщить: к.з могут быть с землей и без нее. На происходящие процессы это не влияет.

О каких же повреждениях шла речь в начале статьи? Как известно, чем выше значение тока, протекающего по участкам цепи, тем больше их нагрев. При достаточной мощности источника при к.з. некоторые участки цепи попросту выгорают, превращаясь в медную пыль (для медных элементов).

Защита от короткого замыкания довольно проста и эффективна. Сообщения о разрушениях из-за замыкания возникают, прежде всего, по причине неправильно подобранных параметров аппаратов защиты, неверной селективности. Если речь идет о бытовой цепи 220 В, то применяют В них при чрезмерном возрастании тока электромагнитный расцепитель, находящийся внутри, разрывает цепь.

Нормальным установившимся режимом работы электроустановки считается такой режим, параметры которого находятся в пределах нормы. Ток короткого замыкания (ток КЗ) возникает при аварии в работе электроустановки. Он чаще всего появляется из-за повреждения изоляции токоведущих частей.

В результате короткого замыкания нарушается бесперебойное питание потребителей, и влечет за собой неисправности и выход из строя оборудования. Вследствие этого при подборе токоведущих элементов и аппаратов необходимо производить их расчет не только для нормальной работы, но и производить проверку по условиям предполагаемого аварийного режима, который может быть вызван коротким замыканием.

Причины повреждения изоляции

  • Воздействие на изоляцию механическим путем.
  • Электрический пробой токоведущих частей вследствие чрезмерных нагрузок или перенапряжения.
  • Подобно нарушению изоляции можно считать причиной повреждения схлестывание неизолированных проводов воздушных линий от сильного ветра.
  • Наброс металлических предметов на линию.
  • Воздействие животных на проводники, находящиеся под напряжением.
  • Ошибки в работе обслуживающего персонала в электроустановках.
  • Сбой в функционировании защит и автоматики.
  • Техническое старение оборудования.
  • Умышленное действие, направленное на повреждение изоляции.

Последствия короткого замыкания

Ток короткого замыкания во много раз превышает ток при нормальной работе оборудования. Возможными последствиями такого замыкания могут быть:

  • Перегрев токоведущих частей.
  • Чрезмерные динамические нагрузки.
  • Прекращение подачи электрической энергии потребителям.
  • Нарушение нормального функционирования других взаимосвязанных приемников, которые подключены к исправным участкам цепи, из-за резкого снижения напряжения.
  • Расстройство системы электроснабжения.

Виды коротких замыканий

Понятие короткого замыкания подразумевает электрическое соединение, которое не предусмотрено условиями эксплуатации оборудования между точками различных фаз, либо нейтрального проводника с фазой или земли с фазой (при наличии контура заземления нейтрали источника питания).

При эксплуатации потребителей напряжение питания может подключаться различными способами:

  • По схеме трехфазной сети 0,4 киловольта.
  • Однофазной сетью (фазой и нолем) 220 В.
  • Источником постоянного напряжения выводами положительного и отрицательного потенциала.

В каждом отдельном случае может возникнуть нарушение изоляции в некоторых точках, вследствие чего возникает ток короткого замыкания.

Для 3-фазной сети переменного тока существуют разновидности короткого замыкания:

  1. Трехфазное замыкание.
  2. Двухфазное замыкание.
  3. Однофазное замыкание на землю.
  4. Однофазное замыкание на землю (Изолированная нейтраль).
  5. Двухфазное замыкание на землю.
  6. Трехфазное замыкание на землю.

При выполнении проекта снабжения электрической энергией предприятия или оборудования подобные режимы требуют определенных расчетов.

Принцип действия короткого замыкания

До начала возникновения короткого замыкания величина тока в электрической цепи имела установившееся значение i п. При резком коротком замыкании в этой цепи из-за сильного уменьшения общего сопротивления цепи электрический ток значительно повышается до значения i к. Вначале, когда время t равно нулю, электрический ток не может резко измениться до другого установившегося значения, так как в замкнутой цепи кроме активного сопротивления R, есть еще и индуктивное сопротивление L. Это увеличивает во времени процесс возрастания тока при переходе на новый режим.

В результате в начальный период короткого замыкания электрический ток сохраняет первоначальное значение iK
= i но. Чтобы ток изменился, необходимо некоторое время. В первые мгновения этого времени ток повышается до максимального значения, далее немного снижается, а затем через определенный период времени принимает установившийся режим.

Период времени от начала замыкания до установившегося режима считается переходным процессом. Ток короткого замыкания можно рассчитать для любого момента в течение переходного процесса.

Ток КЗ при режиме перехода лучше рассматривать в виде суммы составляющих: периодического тока i пt с наибольшей периодической составляющей I пт и апериодического тока i аt (его наибольшее значение – I am).

Апериодическая составляющая тока КЗ во время замыкания постепенно затухает до нулевого значения. При этом ее изменение происходит по экспоненциальной зависимости.

Возможный максимальный ток КЗ считают ударным током i у. Когда нет затухания в начальный момент замыкания, ударный ток определяется:

I у –

i п

m +

i а

t=0 ’,

где
i п m является амплитудой периодической токовой составляющей.

Полезное короткое замыкание

Считается, что короткое замыкание является отрицательным и нежелательным явлением, от которого происходят разрушительные последствия в электроустановках. Оно может создать условия для пожара, отключения защитной аппаратуры, обесточиванию объектов и другим последствиям.

Однако ток короткого замыкания может принести реальную пользу на практике. Есть немало устройств, функционирующих в режиме повышенных значений тока. Для примера можно рассмотреть . Наиболее ярким примером для этого послужит электродуговая сварка, при работе которой накоротко замыкается сварочный электрод с заземляющим контуром.

Такие режимы короткого замыкания действуют кратковременно. Мощность сварочного трансформатора обеспечивает работу при таких значительных перегрузках. Во время сварки в точке соприкосновения электрода возникает очень большой ток. В итоге выделяется значительное количество теплоты, достаточное для расплавления металла в месте касания, и образования сварочного шва достаточной прочности.

Способы защиты

Еще в начале развития электротехники появилась проблема защиты электрических устройств от чрезмерных токовых нагрузок, в том числе и короткого замыкания. Наиболее простым решением стала установка , которые перегорали от их нагревания вследствие превышения тока определенной величины.

Такие плавкие вставки функционируют и в настоящее время. Их основным достоинством является надежность, простота и невысокая стоимость. Однако имеются и недостатки. Простая конструкция предохранителя побуждает человека после сгорания плавкого элемента заменить его самостоятельно подручными материалами в виде скрепок, проволочек и даже гвоздей.

Такая защита не способна обеспечить необходимой защиты от короткого замыкания, так как она не рассчитана на определенную нагрузку. На производстве для отключения цепей, в которых возникло замыкание, используют . Они намного удобнее обычных плавких предохранителей, не требуют замены сгоревшего элемента. После устранения причины замыкания и остывания тепловых элементов, автомат можно просто включить, тем самым подав напряжение в цепь.

Существуют также более сложные системы защиты в виде . Они имеют высокую стоимость. Такие устройства отключают напряжение цепи в случае наименьшей утечки тока. Такая утечка может возникнуть при поражении работника током.

Другим способом защиты от короткого замыкания является токоограничивающий реактор. Он служит для защиты цепей в сетях высокого напряжения, где величина тока КЗ способна достичь такого размера, при котором невозможно подобрать защитные устройства, выдерживающие большие электродинамические силы.

Реактор представляет собой катушку с индуктивным сопротивлением. Он подключен в цепь по последовательной схеме. При нормальной работе на реакторе имеется падение напряжения около 4%. В случае возникновения КЗ основная часть напряжения приходится на реактор. Существует несколько видов реакторов: бетонные, масляные. Каждый из них имеет свои особенности.

Закон Ома при КЗ

В основе расчета замыканий цепи лежит принцип, который определяет вычисление силы тока по напряжению, путем его деления на подключенное сопротивление. Такой же принцип работает и при определении номинальных нагрузок. Отличие в следующем:

  • При возникновении аварийного режима процесс протекает случайным образом, стихийно. Однако он поддается некоторым расчетам по разработанным специалистами методикам.
  • В процессе нормальной работы электрической цепи сопротивление и напряжение находятся в уравновешенном режиме и могут незначительно изменяться в рабочих диапазонах в пределах нормы.

Мощность источника питания

По этой мощности выполняют оценку энергетической силовой возможности разрушительного действия, которое может осуществить ток короткого замыкания, проводят анализ времени протекания, размер.

Для примера рассмотрим, что отрезок медного проводника с площадью сечения 1,5 мм 2 длиной 50 см сначала подсоединили непосредственно к батарее «Крона». А в другом случае этот же кусок провода вставили в бытовую розетку.

В случае с «Кроной» по проводнику будет протекать ток КЗ, который нагреет эту батарею до выхода ее из строя, так как мощности батареи не достаточно для того, чтобы нагреть и расплавить подключенный проводник для разрыва цепи.

В случае с бытовой розеткой сработают защитные устройства. Представим, что эти защиты вышли из строя, и не сработали. В этом случае ток короткого замыкания будет протекать по бытовой проводке, затем по проводке всего подъезда, дома, и далее по воздушной линии или кабеля. Так он дойдет до на подстанции.

В результате к трансформатору подсоединяется длинная цепь с множеством кабелей, проводов, различных соединений. Они намного повысят электрическое сопротивление нашего опытного отрезка провода. Однако даже в таком случае остается большая вероятность того, что этот кусок провода расплавится и сгорит.

Сопротивление цепи

Участок линии электропередач от источника питания до места короткого замыкания обладает некоторым электрическим сопротивлением. Его значение влияет на величину тока короткого замыкания. Обмотки трансформаторов, катушек, дросселей, пластин конденсаторов вносят свой вклад в суммарное сопротивление цепи в виде емкостных и индуктивных сопротивлений. При этом создаются апериодические составляющие, которые искажают симметричность основных форм гармонических колебаний.

Существует множество различных методик, с помощью которых производится расчет ток короткого замыкания. Они позволяют рассчитать с необходимой точностью ток короткого замыкания по имеющейся информации. Практически можно измерить сопротивление имеющейся схемы по методике «фаза-ноль». Это сопротивление делает расчет более точным, вносит соответствующие коррективы при подборе защиты от короткого замыкания.

Что такое короткое замыкание? Чаще всего эту фразу можно услышать от электриков, а также людей, которые вообще не понимают в электронике и электрике. На любой вопрос, почему пошел дым с какого-либо прибора либо устройства, все как один глаголят:» Произошло короткое замыкание». Очень универсальный отмаз для тех, кто желает показаться умным незнайкой).

Природа короткого замыкания

Давайте рассмотрим простейшую цепь, состоящую из лампочки и автомобильного аккумулятора:

В данном случае, по цепи потечет ток и лампочка будет светиться.

Предположим, что наши провода, которые ведут к лампочке, абсолютно голые. Вдруг каким-то чудом на эти проводки падет еще один такой же голый провод. Этот проводок замыкает наши два оголенных провода и начинается самое интересное — в схеме возникает короткое замыкание (КЗ).
Короткое замыкание — это короткий путь для прохождения электрического тока по цепи, где наименьшее сопротивление.

Теперь ток течет и по лампочке, и по проводку. Но у нас проводка намного меньше, чем сопротивление лампочки, и почти весь ток потечет туда, где меньшее сопротивление — то есть по проводку. А так как сопротивление у нашего провода очень мало, то и ток, следовательно, потечет очень большой, согласно Закону Ома . А если потечет большой ток, следовательно, и количество теплоты, выделяемое проводком, будет очень большим, согласно Закону Джоуля-Ленца .
В конце концов, по цепи, которая выделена красным цветом
, будет течь большая и эта цепь будет очень сильно нагреваться. Нагрев проводов может привести к их выгоранию или даже к возгоранию. Этот случай как раз носит название короткого замыкания
.

Вы, наверное, не раз слышали в сводке новостей, что пожар произошел из-за короткого замыкания. В этом случае оголенный провод фазы в каком-то месте задевал оголенный провод нуля, либо фаза задевала землю. Возникало короткое замыкание, и провода стали нагреваться до такой степени, что своим нагревом воспламенили близлежащие предметы. Отсюда пожар.

В основном короткое замыкание происходит в старых домах от старого кабеля, который трещит по швам и может замкнуть между собой. Поэтому, первое, что надо сделать, покупая квартиру либо дом на вторичном рынке — это посмотреть состояние проводки.

Типичные признаки короткого замыкания

  • сгоревшие предохранители в радиоэлектронной аппаратуре (РЭА)
  • нагрев цепи, в которой течет ток короткого замыкания
  • низкое напряжение источника напряжения
  • большой ток
  • дым
  • обугленные провода
  • выгоревшие дорожки печатной платы
  • черный нагар в месте, где произошло КЗ

Как же бороться с коротким замыканием? Это, конечно же, устанавливать предохранители, автоматические выключатели и стараться делать аккуратный монтаж проводки.

Режим короткого замыкания источника — Энциклопедия по машиностроению XXL







Расчёт прочности по предельному состоянию 43 Рафинирование металла шва 22 Редуктор газовый 65, 162 Режим короткого замыкания источника питания дуги 97 Режим нагрузки источника питания дуги 96 Режим сварки 74, 119, 171, 208, 231, 288 Режим холостого хода источника питания дуги 96  [c. 393]

Работу любого источника характеризуют три основных его состояния режим холостого хода (сварочная цепь разомкнута, дуга не горит), режим короткого замыкания (в сварочной цепи течет ток короткого замыкания / .з.) и режим нагрузки (горит дуга при заданном рабочем токе). Этим состояниям соответствуют определенные точки его ВАХ (см. рис. 50).  [c.94]












Такое многократное увеличение тока очень опасно для источника электроэнергии, т. к. выделяемое током тепло пропорционально квадрату тона. По этой причине в момент короткого замыкания разрушается не только изоляция, но расплавляются токонесущие части и провода. Следовательно, режим короткого замыкания является аварийным и недопустимым, /-г напряжение на зажимах источника и=1-Л в момент короткого замыкания падает практически до нуля (очень малая величина). Электродвижущая сила источника в момент короткого замыкания расходуется только на преодоление внутреннего сопротивления источника Е=1г.[c.9]

Источник питания дуги должен быстро реагировать на изменения тока и напряжения в дуге, происходящие в процессе плавления электрода. Это выражается временем восстановления напряжения от нулевого значения в момент короткого замыкания до напряжения повторного зажигания дуги. Это время есть динамическая характеристика источника. Оно не должно превышать 0,05 с иа 25 В. Эта способность трансформатора особенно важна при замыкании каплей переносимого расплавленного металл дугового промежутка, то есть когда, по сути, трансформатор переходит в режим короткого замыкания.  [c.17]

Для питания дуги на участке II с жесткой характеристикой применяют источники с падающей или пологопадающей характеристикой (ручная дуговая сварка, автоматическая под флюсом, сварка в защитных газах неплавящимся электродом). Режим горения дуги определяется точкой пересечения характеристик дуги б и источника тока I (рис. 5.4, б). Точка В соответствует режиму неустойчивого горения дуги, точка С — режиму устойчивого горения дуги (/св и f/д), точка А — режиму холостого хода в работе источника тока в период, когда дуга не горит и сварочная цепь разомкнута. Режим холостого хода характеризуется повышенным напряжением (60. .. 80 В). Точка D соответствует режиму короткого замыкания при зажигании дуги и ее замыкании каплями жидкого электродного металла. Короткое замыкание характеризуется малым напряжением, стремящимся к нулю, и повышенным, но ограниченным током.  [c.225]

При зажигании дуги напряжение между электродом и свариваемым изделием должно быть 60 В, реже 70 В для электродов некоторых марок. Зажигание дуги осуществляют путем кратковременного прикосновения концом электрода к изделию. Существуют два основных способа зажигания клевком и чирком . Первый способ — электрод торцом ударяют в место сварки с небольшим усилием, отводят электрод либо вверх на высоту 4…5 мм, либо вбок и затем приподнимают его на эту высоту. Во втором случае торцом электрода чиркают по месту сварки так, чтобы в конце движения торец был над деталью. Второй способ зажигания используют на только что смененном электроде. Через некоторое время после зажигания дуги на торце электрода образуется козырек (рис. 68). При повторном зажигании электрода козырек нужно разрушить резким ударом торца электрода о свариваемую деталь. Если расстояние между торцом электрода и деталью в момент зажигания дуги будет больше 5…6 мм, то дуга может не возникнуть. В момент отрыва электрода источник питания должен обеспечить быстрый рост напряжения до 20…25 В, что необходимо для возбуждения дуги. После зажигания вести дугу нужно так, чтобы обеспечить проплавление кромок детали и получить требуемое количество наплавленного металла. Для этого нужно поддерживать длину дуги постоянной, равномерно по мере расплавления подавая электрод к изделию. Уменьшение длины дуги ухудшит формирование шва и может вызвать короткое замыкание электро-  [c.116]












По проводнику с небольшим сопротивлением и далее по проводам электрической цепи будет протекать такой силы ток, который вызовет выделение большого количества тепла, приведет к обугливанию и сгоранию изоляции проводов, расплавлению материала проводов, порче электроизмерительных приборов, оплавлению контактов выключателей, ножей рубильников, повреждению источника питания электрической цепи. Аварийный режим работы сети, когда вследствие уменьшения ее сопротивления ток в цепи резко увеличивается против нормального, называется коротким замыканием. Ввиду опасных и разрушительных последствий короткого замыкания необходимо, чтобы соблюдались следующие условия  [c.25]

Источники тока для питания сварочной дуги могут иметь различные внешние характеристики (рис. 194, а) падающую 1, пологую 2, жесткую 3 и возрастающую 4. Внешней характеристикой источника называется зависимость напряжения на его выходных клеммах от тока в цепи при нагрузке. Источник сварочного тока выбирают в зависимости от вольтамперной характеристики дуги, соответствующей применяемому способу сварки. Для питания дуги с жесткой характеристикой требуются источники сварочного тока с падающей внешней характеристикой. Режим горения сварочной дуги определяется точкой пересечения характеристик дуги 1 и источника тока 2 (рис. 194, б). Точка С на рис. 194, б является точкой устойчивого горения дуги. Последнее определяется тем, что после случайного отклонения режим горения дуги восстанавливается. Случайное увеличение тока, соответствующего точке С, приведет к уменьшению напряжения источника питания, что после окончания действия случайной отклоняющей причины повлечет за собой уменьшение тока, т. е. восстановление режима устойчивого горения дуги. При случайном уменьшении тока все параметры изменяются в обратном порядке и в конечном итоге также происходит восстановление устойчивого режима горения дуги. Точка В на том же рисунке соответствует неустойчивому горению дуги. При изменении соответствующего ей тока дуга либо гаснет, либо ток дуги начинает возрастать до тех пор пока дуга достигнет режима устойчивого горения. Характерными точками внешней характеристики источника являются точки А п О. Точка А соответствует режиму холостого хода в работе источника питания в период, когда дуга не горит и сварочная цепь разомкнута. Режим холостого хода характеризуется повышенным напряжением (60—80 в). Точка О соответствует режиму короткого замыкания, который имеет место при зажигании дуги и замыкании дуги каплями жидкого электродного металла. Короткое замыкание характеризуется малым значением напряжения, стремящегося к нулю, и повышенной величиной тока, однако,  [c.302]

Кроме необходимых статических характеристик, источник питания должен обладать оптимальными динамическими свойствами. При сварке плавящимся электродом возбуждение дуги и перенос капель с электрода на изделие связаны с замыканиями дугового промежутка, в некоторых случаях с ее угасанием и повторным зажиганием после разрыва капли. Поэтому источник питания работает в условиях резкого изменения режима холостой ход — короткое замыкание (первоначальное возбуждение дуги) — рабочий режим (горение дуги) — короткое замыкание (переход капель) — рабочий режим и т. д.  [c.604]

Стопорным режимом называется режим интенсивного торможения выходного звена машинного агрегата, приводящий к практически мгновенной его остановке. При этом ротор двигателя останавливается за весьма малый промежуток времени, нагружая приводной механизм значительным по величине крутящим моментом. Если в цепи, управления двигателем предусмотрено реле, ограничивающее максимальную величину тока, а быстродействие коммутирующей аппаратуры достаточное, то отключение двигателя от источника питания может произойти до остановки ротора. При недостаточном быстродействии коммутирующей аппаратуры или при отсутствии максимальной защиты двигателя последний оказывается включенным до остановки ротора, т. е. работает в так называемом режиме короткого замыкания.  [c.308]












Перегрузки бывают кратковременные и длительные. Кратковременные перегрузки наступают при подключении и отключении потребителя и при включении и отключении стабилизатора от питающего источника, а также при случайном, быстро прекращающемся, коротком замыкании на выходе стабилизатора. Длительные перегрузки чаще всего являются следствием короткого замыкания на выходе стабилизатора и реже при его холостом ходе.  [c.272]

Измерение У-параметров биполярного транзистора проводятся с помощью специальных схем, пример одной из которых приведен на рис. П13. Два источника напряжения обеспечивают заданный режим по постоянному току, короткое замыкание на выходе по переменному току и подачу на вход источника ЭДС для проведения измерений модулей, действительных и мнимых частей этих У-параметров.  [c.364]

Главными компонентами установки для вакуумного эле тродугового переплава являются источник энергии, тигел] ный агрегат, вакуумные насосы и система управления [5]. качестве источников электроэнергии могут быть использовг ны и дроссели насыщения, и кремниевые выпрямители. В лк бом случае цель заключается в том, чтобы обеспечить раб( ту печи на максимальном желаемом энергетическом уровн при непременно стабильных характеристиках дуги. Очен часто плавление развивается в условиях близких или соо ветствующих режиму короткого замыкания, и система должн быть способна устранить этот мгновенно возникший режи короткого замыкания, восстановить дугу и вернуть к дейст ВИЮ заданный режим работы агрегата.  [c.136]

Источники питания имеют различные внещние вольт-амперные характеристики (рис. 92) естественную, жесткую и щтыковую. Источники питания с естественной 1 и жесткой 2 характеристиками являются источниками напряжения. Для них режим короткого замыкания является аварийным, поскольку их внутреннее сопротивление близко к нулю. Источники питания со штыковой 3 характеристикой являются источниками тока. Для источников тока параметрического типа аварийным является режим холостого хода, так как они содержат реактивные элементы, напряжение на которых при отключении нагрузки резко возрастает, что может вызвать пробой отдельных элементов выпрямительного агрегата. В статических преобразователях, используемых при размерной ЭХО, применяются неуправляемые и управляемые вентильные схемы.  [c.158]

Источник питания должен быть рассчитан на работу в режим короткого замыкания и иметь устройства для регулирования токг дуги.  [c.444]

В качестве источника тока при воздушно-дуговой резке можно использовать обычные сварочные генераторы постоянного тока с крутопадающей или пологопадаюшей характеристикой. Важно, чтобы источник тока обладал достаточной мощностью, обеспечивающей резку мощными дугами, поскольку интенсивность выплавления металла при воздушно-дуговой резке прямо пропорциональна величине рабочего тока. В то же время напряжение на режущем разряде выше, чем напряжение сварочной дуги. В связи с этим напряжение холостого хода источника тока не должно быть ниже 65— 70 в. Источник тока должен обладать хорошими динамическими свойствами, поскольку при воздушно-дуговой резке режим короткого замыкания возникает чаще, чем при дуговой электросварке. Режим повторного включения при воздущно-дуговой резке достигает 80%, в то время как при сварке покрытыми электродами — только 50%.  [c.48]

Пусть р-п-переход расположен вблизи от освещаемой поверхности полупроводника. При использовании солнечного элемента в качестве источника электроэнергии к его выводам должно бьпь подсоединено сог эотивление нагрузки. Рассмотрим вначале два 1файних случая =0 (режим короткого замыкания) и = со (режим холостого хода). Зонные диаграммы для  [c.14]

Пересечение внешней характеристики с осью ординат Uопределяет напряжение холостого хода источника U , а с осью абсцисс / — силу тока короткого замыкания 4 при замыкании электрода на изделие и замыкании каплями жидкого электронного металла. Короткое замыкание характеризуется малым напряжением, стремящимся к нулю, и повышенным, но ограниченным током. Режим холостого хода, когда сварочная цепь разомкнута и дуга не горит, характеризуется повышенным напряжением (60…70 В). Рабочее напряжение дуги поддерживается в пределах 16…30 В.  [c.379]

При проведении электрических стендовых испытаний источников питания измерения производят измерительными приборами класса не ниже 0,5 при государственных испытаниях и не ниже 1,5 при приемо-сдаточных. Во всех случаях снимаются внешние статические характеристики или их характерные точки, в частности, значения напряжения холостого хода и силы тока при нормированном рабочем напряжении. Изоляцию силовых развязывающих трансформаторов испытывают на сопротивление и электрическую прочность между обмотками, а также между каждой обмоткой и корпусом. Прочность проверяют повышенным переменным напряжением 2. .. 4 кВ, а межвит-ковую прочность — двойным (к номинальному напряжению) при повышенной частоте 100. .. 400 Гц. Источники питания, режим работы которых предполагает или допускает короткие замыкания нагрузок, испытывают на прочность единичными кратковременными, имитирующими замыкания, нагрузками с нормированным сопротивлением (обычно 10 МОм).  [c.48]












НЫХ процессов (скорость выхода на рабочий режим после вынуж денных коротких замыканий и т. д.) и определяются электромаг нитной инерцией источника. Чем выше динамические свойств источника, тем стабильнее процесс сварки.  [c.444]

Для питания сварочной дуги требуется источник тока, удовлетворяющий ряду требований. Он должен иметь достаточно высокое напряжение холостого хода, т. е. обеспечивать напряжение между электродом и изделием, достаточное для легкого возбуждения дуги, по не превышающее 80—90 В, что определяется нормами безопасности труда. Источник должен обладать достаточной мощностью для выполнения определенных сварочных работ. Сварочный источник не должен выходить из строя в режиме короткого замыкания, поскольку этот режим всегда имеет место при сварке. Он долл4ен иметь устройство для возможности плавного регулнрован]1Я сварочного тока, обладать хорошими динамическими свойствами, т. е. обеспечивать быстрое восстановлени е режима после коротких замыканий и устойчиво работать на зада ном режиме.  [c.377]

Системы многопостового питания в этих случаях должны быть различными. Если сварку выполняют на режимах с частыми замыканиями разрядного промежутка, в цепь каждого сварочного поста необходимо включать индуктивность, которая уменьшает влияние одного поста на другие (рис. 25, а) значение постовой индуктивности при сварке проволоками 00,8—1,4 мм выбирают равным 0,2—0,25 мГн. При выполнении сварки ра различающихся режимах напряжение холостого хода источника питания устанавливают по максимальному необходимому напряжению сварки, а напряжение на постах настраивают малогабаритными балластными реостатами (рис. 25, б). Если сварка выполняется на режимах без коротких замыканий или с редкими корют-кими замыканиями, то индуктивность в цепь поста не включают. При сварке на режимах с короткими замыканиями и без них на отдельных постах может оказаться более выгодным устройство многопостовой системы с двумя отдельными шинопроводами на различное напряжение, питаемыми от отдельных источников. При двухдуговой сварке на одинаковых режимах режим регулируют одновременно на обеих дугах изменением напряжени я источника питания. В этом  [c.65]

Скольжение 8=1, когда ротор электродвигателя неподвижен при включенной обмотке статора. Такой режим называется режимом короткого замыкания электродви а-теля. Пуск асинхронного электродвигателя всегда начинается именно с этого режима. Под действием постороннего источника механической энергии, например опускаемого груза, ротор электродвигателя может вращаться с частотой больше синхронной. В этом случае скольженне считается отрицательным.  [c.168]


Что происходит при коротком замыкании. Короткое замыкание. Что это такое, и какие замыкания бывают

Любой человек, чья работа связана с обслуживанием электротехники, очень хорошо знает о тех неприятностях, которые таит в себе короткое замыкание (к.з.). Иногда считается, что оно представляет собой повреждение. Это не так. Короткое замыкание — это процесс, или, если угодно, аварийный режим работы какого-либо участка электроустановки. А вот последствия его действительно приводят к повреждениям. Общепринятое определение гласит: «Короткое замыкание — это непосредственное соединение двух или более точек электрической цепи, обладающих различным потенциалом. Является ненормальным (непредусмотренным) режимом работы».

Чтобы понять, что именно происходит в цепи в тот момент, когда там возникает короткое замыкание, необходимо вспомнить принципы функционирования элементов контура. Представим простейшую цепь, состоящую из двух проводников и нагрузки (например, лампочки). В обычных условиях в проводнике существует направленное движение заряженных элементарных частиц, обусловленное постоянным воздействием источника. Они перемещаются от одного полюса источника к другому через два участка провода и лампу. Соответственно, лампа излучает свет, так как частицы совершают в ней определенную работу.

При направление движения постоянно изменяется, но в данном случае это не принципиально. Количество электронов, проходящих по определенному участку цепи за единицу времени, ограничивается сопротивлением лампы, проводников, источника ЭДС. Другими словами, ток не растет бесконечно, а соответствует установившемуся режиму.

Но вот по какой-либо причине повреждается изоляция на участке цепи. К примеру, лампу залило водой. В этом случае ее уменьшается. В результате текущий по контуру ток ограничивается суммарным сопротивлением источника питания, проводов и водного «перешейка» на лампе. Обычно эта сумма настолько ничтожна, что в расчетах не учитывается (исключение составляют специализированные вычисления).

Итогом является практически бесконечный рост тока, определяемого по классическому закону Ома. В данном случае часто упоминают мощность короткого замыкания. Она определяется предельным значением электрического тока, который способен выдать источник питания до выхода из строя. Кстати, именно поэтому запрещается соединять проводком (закорачивать) противоположные контакты батареек.

Хотя в примере мы рассматриваем устранение из цепи сопротивления лампы вследствие попадания на нее воды, причин короткого замыкания множество. К примеру, если говорить об этой же схеме, то к.з. также может возникнуть, если будет нарушена изоляция хотя бы одного провода и он соприкоснется с землей. В этом случае ток от источника питания последует по пути наименьшего сопротивления, то есть в землю, обладающую огромной емкостью. Повреждение изоляции сразу двух проводов и их соприкосновение приведет к тому же самому результату.

Вышесказанное можно обобщить: к.з могут быть с землей и без нее. На происходящие процессы это не влияет.

О каких же повреждениях шла речь в начале статьи? Как известно, чем выше значение тока, протекающего по участкам цепи, тем больше их нагрев. При достаточной мощности источника при к.з. некоторые участки цепи попросту выгорают, превращаясь в медную пыль (для медных элементов).

Защита от короткого замыкания довольно проста и эффективна. Сообщения о разрушениях из-за замыкания возникают, прежде всего, по причине неправильно подобранных параметров аппаратов защиты, неверной селективности. Если речь идет о бытовой цепи 220 В, то применяют В них при чрезмерном возрастании тока электромагнитный расцепитель, находящийся внутри, разрывает цепь.

Наверняка многие слышали такое словосочетание как короткое замыкание, но мало кто понимает, из-за чего возникает данное явление, чем оно опасно и какие процессы происходят во время КЗ. В этой статье мы подробно рассмотрим данный вопрос, так как «коротыш в проводке» — это достаточно частая ситуация, которая является очень опасной и может привести к неблагоприятным последствиям. Итак, причины возникновения короткого замыкания, способы предотвращения и последствия мы рассмотрели ниже.


Что это такое?

Электрическая цепь — это, как правило, два проводника с разноименным потенциалом и подключенным потребителем тока. Каждый конечный потребитель имеет свое внутреннее сопротивление, которое сопротивляется току и ограничивает, тем самым дозируя его количество и плотность в проводнике, заставляя производить работу.

В момент, когда сопротивление резко уменьшается до статической погрешности сопротивления проводников, электрический ток, ничем практически не ограниченный, возрастает до такой величины, что сечение проводников становится малым и проходя через них, разогревает жилы до температуры разрушения и плавления. Поэтому частый спутник короткого замыкания — это огонь, расплавленный металл проводников и вспомогательных механизмов.

Признаками замыкания в проводке являются запах гари, искрение и возгорание проводов, а также отключение электричества на определенном участке или же во всей сети.

Как возникает КЗ?

Итак, рассмотрим основные причины возникновения короткого замыкания в электропроводке и электроустановках.

Высокое напряжение
. В момент выше допустимых параметров, присутствует возможность электрического пробоя изоляции проводника или электрической схемы. В результате развивается утечка тока до размеров КЗ, с созданием кратковременного стабильного дугового разряда.

Старая изоляция
. Жилые и промышленные фонды, не проводившие замену электрической проводки — это первые претенденты на спонтанные КЗ. Любая изоляция, используемая в электропроводке, имеет свой ресурс. Со временем она разрушается под воздействием внешних факторов, что и приводит к возникновению замыкания.

Внешнее механическое воздействие.
Снятие изоляции с провода, ее перетирание и прочее воздействие на защитную оболочку, ослабляющее ее свойства, рано или поздно вызовут возгорание и КЗ. К примеру, в быту часто причиной возникновения короткого замыкания является повреждение проводки при сверлении стен. О том, читайте в нашей статье.

Посторонние предметы
. Сюда относится пыль различного происхождения, мелкие животные, детали с соседних узлов, волей случая попавших на электрические проводники, вызвав и развив таким образом КЗ.

Прямой удар молнии.
Происходит тоже, что и при (смотри выше).

Пример последствия от возникновения КЗ в электроустановке демонстрируется на видео:

Последствия короткого замыкания — это выгоревшие участки проводки и ее возгорание!

Виды явлений

Самое распространенное — это замыкание на землю, когда либо одна фаза взаимодействует с землей, либо две фазы взаимодействует с землей, на одном или нескольких участках. Короткое замыкание на землю, встречается в системах с глухозаземленной нейтралью и составляют до 70% всех случаев.

Существует также межфазное КЗ, когда происходит взаимодействие двух фаз между собой. Происходит в следствии нарушении изоляции в трехфазном оборудовании.

Ну и последний вид КЗ — трехфазное, когда взаимодействуют все три фазы. На схеме ниже изображены основные виды коротких замыканий:

Способы предотвращения

Для предотвращения развития КЗ и защиты электрических устройств и линий электроснабжения самым эффективным методом является или же плавких предохранителей. Автомат (на фото ниже) при возникновении «коротыша» своевременно отключит питание, тем самым предотвратит возникновение опасной ситуации.

Еще один способ предотвратить возникновение короткого замыкания — своевременная , благодаря которой можно визуально определить место оплавления изоляции и перейти к устранению неполадки.

Добрый день, уважаемые читатели сайта «Заметки электрика».

Давно хотел написать статью про короткое замыкание. Но все как то не доходили руки.

Сегодня решился, потому как повлияли на меня последние события, произошедшие на распределительной подстанции нашего предприятия.

Ранее в статьях мы говорили, что вызывают короткие замыкания, или сокращенно, к.з.

Короткое замыкание — это одно из самых тяжелых и опасных видов повреждения.

Вы спросите почему? Читайте ниже.

Что же такое короткое замыкание?

Википедия на этот вопрос отвечает, что короткое замыкание — это:

Определение прочитали.

А теперь давайте рассмотрим подробно, что же происходит с параметрами электроустановки в момент короткого замыкания.

При возникновении короткого замыкания, напряжение на источнике питания, а правильнее назвать ЭДС, замыкается «накоротко» через небольшое (малой величины) сопротивление кабельных и воздушных линий, обмоток трансформаторов и генераторов. Отсюда и название «короткое замыкание».

В «накоротко» замкнутой цепи появляется ток очень большой величины, который и называется током короткого замыкания.

Рассмотрим классификацию коротких замыканий.

Короткие замыкания разделяются по количеству замкнувшихся фаз:

  • трехфазные короткие замыкания
  • двухфазные короткие замыкания
  • однофазные короткие замыкания

Короткие замыкания разделяются по замыканию:

  • с землей
  • без земли

Короткие замыкания разделяются по количеству замкнувшихся точек в сети:

  • в одной точке
  • в двух точках
  • в нескольких точках (более двух)

Пример

Рассмотрим пример.

Допустим, что наш потребитель питается с подстанции через воздушную линию (ВЛ) электропередач. Питающая линия является транзитной, поэтому питание потребителя осуществляется отпайкой от линии ВЛ в точке «О».

Пунктирной линией под номером 2 показан уровень напряжения на протяжении всей воздушной линии до возникновения короткого замыкания.

По рисунку видно, что напряжение в любой точке электрической сети равно разнице ЭДС источника питания и падения напряжения в электрической цепи до необходимой нам точки.

Например, напряжение в точке «О» можно рассчитать по формуле:

Uо = E — I*Zo, где

  • E — ЭДС источника питания, в нашем случае генератора
  • Zo — полное сопротивление воздушной линий от источника питания до точки «О» (состоит из активного и реактивного сопротивления)
  • I — ток, протекающий по воздушной линии в данный момент времени.

Предположим, что по каким-либо причинам произошло короткое замыкание на воздушной линии, но за пределами нашей отпайки. Назовем эту точку короткого замыкания буквой «К».

Что же произойдет в момент короткого замыкания?

В момент короткого замыкания по воздушной линии проходит уже не номинальный ток, а ток короткого замыкания большой величины, поэтому возрастает падение напряжения на каждом элементе электрической цепи. А именно на сопротивлении Zo и Zк.

Самое наибольшее снижение напряжения будет в месте короткого замыкания, т. е. в точке «К». В остальных точках воздушной линии, удаленных от места к.з., напряжение снизится чуть меньше (это видно на рисунке — линия под номером 1).

В одной из своих статей я привел наглядный . Переходите по ссылочке и знакомьтесь с материалами.

Последствия от короткого замыкания

Мы уже выяснили, что в момент короткого замыкания происходит резкое увеличение величины тока и снижение напряжения, что приводит к следующим последствиям.

1. Разрушения

Вспомним немного физику.

По закону известного физика Джоуля-Ленца, ток короткого замыкания, протекая по активному сопротивлению электрической цепи в течение некоторого времени, выделяет в нем тепло, которое рассчитывается по формуле:

В точке короткого замыкания это тепло, а также пламя электрической дуги, производят огромные разрушения. И чем больше ток короткого замыкания и время его прохождения по цепи, тем больше будут разрушения.

Чтобы было понятно Вам насколько эти разрушения масштабны, ниже приведу примеры из своей практики.

Привод переключающего устройства РПН. Короткое замыкание произошло в обмотке асинхронного двигателя

2. Повреждение изоляции

Во время прохождения тока короткого замыкания по неповрежденным линиям, происходит их нагрев выше предельной допустимой температуры, что приводит к повреждению их изоляции.

Активная часть трансформатора. Короткое замыкание произошло по причине повреждения изоляции

Короткое замыкание кабеля. Последствия

3. Потребители и электроприемники

Снижение напряжения при коротком замыкании нарушает нормальную работу потребителей и электроприемников .

Например, асинхронный при снижении напряжения сети может вообще остановиться, т.к. момент его вращения может оказаться меньше момента сопротивления и трения механизмов.

Также нарушается нормальная работа и осветительных остановок. Здесь я думаю объяснять не требуется.

Смотрите наглядное видео про причины и последствия короткого замыкания в электроустановке 400 (В) на одной из наших подстанций:

А вот уже случай по-серьезнее — трехфазное короткое замыкание в сети 10 (кВ).

Вот еще фрагменты аварии, которая возникла по причине короткого замыкания в разделке кабеля 10 (кВ):

P.S. В завершении статьи на тему короткое замыкание, хочется подтвердить сказанное в начале своей статьи, что короткое замыкание является самым опасным и тяжелым видом повреждения, которое требует мгновенного и быстрого реагирования и отключения поврежденного участка цепи.

Нормальным установившимся режимом работы электроустановки считается такой режим, параметры которого находятся в пределах нормы. Ток короткого замыкания (ток КЗ) возникает при аварии в работе электроустановки. Он чаще всего появляется из-за повреждения изоляции токоведущих частей.

В результате короткого замыкания нарушается бесперебойное питание потребителей, и влечет за собой неисправности и выход из строя оборудования. Вследствие этого при подборе токоведущих элементов и аппаратов необходимо производить их расчет не только для нормальной работы, но и производить проверку по условиям предполагаемого аварийного режима, который может быть вызван коротким замыканием.

Причины повреждения изоляции

  • Воздействие на изоляцию механическим путем.
  • Электрический пробой токоведущих частей вследствие чрезмерных нагрузок или перенапряжения.
  • Подобно нарушению изоляции можно считать причиной повреждения схлестывание неизолированных проводов воздушных линий от сильного ветра.
  • Наброс металлических предметов на линию.
  • Воздействие животных на проводники, находящиеся под напряжением.
  • Ошибки в работе обслуживающего персонала в электроустановках.
  • Сбой в функционировании защит и автоматики.
  • Техническое старение оборудования.
  • Умышленное действие, направленное на повреждение изоляции.

Последствия короткого замыкания

Ток короткого замыкания во много раз превышает ток при нормальной работе оборудования.

Возможными последствиями такого замыкания могут быть:

  • Перегрев токоведущих частей.
  • Чрезмерные динамические нагрузки.
  • Прекращение подачи электрической энергии потребителям.
  • Нарушение нормального функционирования других взаимосвязанных приемников, которые подключены к исправным участкам цепи, из-за резкого снижения напряжения.
  • Расстройство системы электроснабжения.

Виды коротких замыканий

Понятие короткого замыкания подразумевает электрическое соединение, которое не предусмотрено условиями эксплуатации оборудования между точками различных фаз, либо нейтрального проводника с фазой или земли с фазой (при наличии контура заземления нейтрали источника питания).

При эксплуатации потребителей напряжение питания может подключаться различными способами:

В каждом отдельном случае может возникнуть нарушение изоляции в некоторых точках, вследствие чего возникает ток короткого замыкания.

Для 3-фазной сети переменного тока существуют разновидности короткого замыкания:

  1. Трехфазное замыкание.
  2. Двухфазное замыкание.
  3. Однофазное замыкание на землю.
  4. Однофазное замыкание на землю (Изолированная нейтраль).
  5. Двухфазное замыкание на землю.
  6. Трехфазное замыкание на землю.

При выполнении проекта снабжения электрической энергией предприятия или оборудования подобные режимы требуют определенных расчетов.

Принцип действия короткого замыкания

До начала возникновения короткого замыкания величина тока в электрической цепи имела установившееся значение i п. При резком коротком замыкании в этой цепи из-за сильного уменьшения общего сопротивления цепи электрический ток значительно повышается до значения i к. Вначале, когда время t равно нулю, электрический ток не может резко измениться до другого установившегося значения, так как в замкнутой цепи кроме активного сопротивления R, есть еще и индуктивное сопротивление L. Это увеличивает во времени процесс возрастания тока при переходе на новый режим.

В результате в начальный период короткого замыкания электрический ток сохраняет первоначальное значение iK
= i но. Чтобы ток изменился, необходимо некоторое время. В первые мгновения этого времени ток повышается до максимального значения, далее немного снижается, а затем через определенный период времени принимает установившийся режим.

Период времени от начала замыкания до установившегося режима считается переходным процессом. Ток короткого замыкания можно рассчитать для любого момента в течение переходного процесса.

Ток КЗ при режиме перехода лучше рассматривать в виде суммы составляющих: периодического тока i пt с наибольшей периодической составляющей I пт и апериодического тока i аt (его наибольшее значение – I am).

Апериодическая составляющая тока КЗ во время замыкания постепенно затухает до нулевого значения. При этом ее изменение происходит по экспоненциальной зависимости.

Возможный максимальный ток КЗ считают ударным током i у. Когда нет затухания в начальный момент замыкания, ударный ток определяется:

I у –

i п

m +

i а

t=0 ’,

где
i п m является амплитудой периодической токовой составляющей.

Полезное короткое замыкание

Считается, что короткое замыкание является отрицательным и нежелательным явлением, от которого происходят разрушительные последствия в электроустановках. Оно может создать условия для пожара, отключения защитной аппаратуры, обесточиванию объектов и другим последствиям.

Однако ток короткого замыкания может принести реальную пользу на практике. Есть немало устройств, функционирующих в режиме повышенных значений тока. Для примера можно рассмотреть . Наиболее ярким примером для этого послужит электродуговая сварка, при работе которой накоротко замыкается сварочный электрод с заземляющим контуром.

Такие режимы короткого замыкания действуют кратковременно. Мощность сварочного трансформатора обеспечивает работу при таких значительных перегрузках. Во время сварки в точке соприкосновения электрода возникает очень большой ток. В итоге выделяется значительное количество теплоты, достаточное для расплавления металла в месте касания, и образования сварочного шва достаточной прочности.

Способы защиты

Еще в начале развития электротехники появилась проблема защиты электрических устройств от чрезмерных токовых нагрузок, в том числе и короткого замыкания. Наиболее простым решением стала установка , которые перегорали от их нагревания вследствие превышения тока определенной величины.

Такие плавкие вставки функционируют и в настоящее время. Их основным достоинством является надежность, простота и невысокая стоимость. Однако имеются и недостатки. Простая конструкция предохранителя побуждает человека после сгорания плавкого элемента заменить его самостоятельно подручными материалами в виде скрепок, проволочек и даже гвоздей.

Такая защита не способна обеспечить необходимой защиты от короткого замыкания, так как она не рассчитана на определенную нагрузку. На производстве для отключения цепей, в которых возникло замыкание, используют . Они намного удобнее обычных плавких предохранителей, не требуют замены сгоревшего элемента. После устранения причины замыкания и остывания тепловых элементов, автомат можно просто включить, тем самым подав напряжение в цепь.

Существуют также более сложные системы защиты в виде . Они имеют высокую стоимость. Такие устройства отключают напряжение цепи в случае наименьшей утечки тока. Такая утечка может возникнуть при поражении работника током.

Другим способом защиты от короткого замыкания является токоограничивающий реактор. Он служит для защиты цепей в сетях высокого напряжения, где величина тока КЗ способна достичь такого размера, при котором невозможно подобрать защитные устройства, выдерживающие большие электродинамические силы.

Реактор представляет собой катушку с индуктивным сопротивлением. Он подключен в цепь по последовательной схеме. При нормальной работе на реакторе имеется падение напряжения около 4%. В случае возникновения КЗ основная часть напряжения приходится на реактор. Существует несколько видов реакторов: бетонные, масляные. Каждый из них имеет свои особенности.

Закон Ома при КЗ

В основе расчета замыканий цепи лежит принцип, который определяет вычисление силы тока по напряжению, путем его деления на подключенное сопротивление. Такой же принцип работает и при определении номинальных нагрузок. Отличие в следующем:

  • При возникновении аварийного режима процесс протекает случайным образом, стихийно. Однако он поддается некоторым расчетам по разработанным специалистами методикам.
  • В процессе нормальной работы электрической цепи сопротивление и напряжение находятся в уравновешенном режиме и могут незначительно изменяться в рабочих диапазонах в пределах нормы.

Мощность источника питания

По этой мощности выполняют оценку энергетической силовой возможности разрушительного действия, которое может осуществить ток короткого замыкания, проводят анализ времени протекания, размер.

Для примера рассмотрим, что отрезок медного проводника с площадью сечения 1,5 мм 2 длиной 50 см сначала подсоединили непосредственно к батарее «Крона». А в другом случае этот же кусок провода вставили в бытовую розетку.

В случае с «Кроной» по проводнику будет протекать ток КЗ, который нагреет эту батарею до выхода ее из строя, так как мощности батареи не достаточно для того, чтобы нагреть и расплавить подключенный проводник для разрыва цепи.

В случае с бытовой розеткой сработают защитные устройства. Представим, что эти защиты вышли из строя, и не сработали. В этом случае ток короткого замыкания будет протекать по бытовой проводке, затем по проводке всего подъезда, дома, и далее по воздушной линии или кабеля. Так он дойдет до на подстанции.

В результате к трансформатору подсоединяется длинная цепь с множеством кабелей, проводов, различных соединений. Они намного повысят электрическое сопротивление нашего опытного отрезка провода. Однако даже в таком случае остается большая вероятность того, что этот кусок провода расплавится и сгорит.

Сопротивление цепи

Участок линии электропередач от источника питания до места короткого замыкания обладает некоторым электрическим сопротивлением. Его значение влияет на величину тока короткого замыкания. Обмотки трансформаторов, катушек, дросселей, пластин конденсаторов вносят свой вклад в суммарное сопротивление цепи в виде емкостных и индуктивных сопротивлений. При этом создаются апериодические составляющие, которые искажают симметричность основных форм гармонических колебаний.

Существует множество различных методик, с помощью которых производится расчет ток короткого замыкания. Они позволяют рассчитать с необходимой точностью ток короткого замыкания по имеющейся информации. Практически можно измерить сопротивление имеющейся схемы по методике «фаза-ноль». Это сопротивление делает расчет более точным, вносит соответствующие коррективы при подборе защиты от короткого замыкания.

Однажды одной даме, не очень сведущей в электротехнике, монтер сообщил причину пропадания света в ее квартире. Это оказалось короткое замыкание, и женщина потребовала немедленно его удлинить. Над этой историей можно посмеяться, но лучше все же рассмотреть эту неприятность подробнее. Специалистам-электрикам и без этой статьи известно, что это за явление, чем оно грозит и как рассчитать ток короткого замыкания. Изложенная ниже информация адресована людям, не имеющим технического образования, но, как и все прочие, не застрахованным от неприятностей, связанных с эксплуатацией техники, машин, производственного оборудования и самых обычных бытовых приборов. Каждому человеку важно знать, что такое короткое замыкание, каковы его причины, возможные последствия и методы его предотвращения. Не обойтись в этом описании и без знакомства с азами электротехнической науки. Не знающий их читатель может заскучать и не дочитать статью до конца.

Популярное изложение закона Ома

Независимо от того, каков характер тока электрической цепи, он возникает только в том случае, если существует разница потенциалов (или напряжение, это то же самое). Природа этого явления может быть объяснена на примере водопада: если есть разность уровней, вода течет в каком-то направлении, а когда нет — она стоит на месте. Даже школьникам известен закон Ома, согласно которому, ток тем больше, чем выше напряжение, и тем меньше, чем выше сопротивление, включенное в нагрузку:

I — величина тока, которую иногда называют «силой тока», хотя это не совсем грамотный перевод с немецкого языка. Измеряется в Амперах (А).

На самом деле силой (то есть причиной ускорения) ток сам по себе не обладает, что как раз и проявляется во время короткого замыкания. Этот термин уже стал привычным и употребляется часто, хотя преподаватели некоторых вузов, услышав из уст студента слова «сила тока» тут же ставят «неуд». «А как же огонь и дым, идущие от проводки во время короткого замыкания? — спросит настырный оппонент, — Это ли не сила?» Ответ на это замечание есть. Дело в том, что идеальных проводников не существует, и нагрев их обусловлен именно этим фактом. Если предположить, что R=0, то и тепло бы не выделялось, как ясно из закона Джоуля-Ленца, приведенного ниже.

U — та самая разница потенциалов, называемая также напряжением. Измеряется в Вольтах (у нас В, за границей V). Его также называют электродвижущей силой (ЭДС).

R — электрическое сопротивление, то есть способность материала препятствовать прохождению тока. У диэлектриков (изоляторов) оно большое, хотя и не бесконечное, у проводников — малое. Измеряется в Омах, но оценивается в качестве удельной величины. Само собой, что чем толще провод, тем он лучше проводит ток, а чем он длиннее, тем хуже. Поэтому удельное сопротивление измеряется в Омах, умноженных на квадратный миллиметр и деленных на метр. Кроме этого, на его величину влияет температура, чем она выше, тем больше сопротивление. Например, золотой проводник длиной в 1 метр и сечением в 1 кв. мм при 20 градусах Цельсия обладает общим сопротивлением 0,024 Ома.

Есть еще формула закона Ома для полной цепи, в нее введено внутреннее (собственное) сопротивление источника напряжения (ЭДС).

Две простых, но важных формулы

Понять причину, по которой возникает ток короткого замыкания, невозможно без усвоения еще одной нехитрой формулы. Мощность, потребляемая нагрузкой, равна (без учета реактивных составляющих, но о них позже) произведению тока на напряжение.

P — мощность, Ватт или Вольт-Ампер;

U — напряжение, Вольт;

I — ток, Ампер.

Мощность бесконечной не бывает, она всегда чем-то ограничена, поэтому при ее фиксированной величине при увеличении тока напряжение уменьшается. Зависимость этих двух параметров рабочей цепи, выраженная графически, называется вольт-амперной характеристикой.

И еще одна формула, необходимая для того, чтобы произвести расчет токов короткого замыкания, это закон Джоуля-Ленца. Она дает представление о том, сколько тепла выделяется при сопротивлении нагрузке, и очень проста. Проводник будет греться с интенсивностью, пропорциональной величинам напряжения и квадрата тока. И, конечно же, формула не обходится без времени, чем дольше раскаляется сопротивление, тем больше оно выделит тепла.

Что происходит в цепи при коротком замыкании

Итак, читатель может считать, что освоил все главные физические закономерности для того, чтобы разобраться в том, какой может быть величина (ладно, пусть будет сила) тока короткого замыкания. Но сначала следует определиться с вопросом о том, что, собственно, это такое. КЗ (короткое замыкание) — это ситуация, при которой сопротивление нагрузки близко к нулю. Смотрим на формулу закона Ома. Если рассматривать его вариант для участка цепи, несложно понять, что ток будет стремиться к бесконечности. В полном варианте он будет ограничен сопротивлением источника ЭДС. В любом случае ток короткого замыкания очень велик, а по закону Джоуля-Ленца, чем он больше, тем сильнее греется проводник, по которому он идет. Причем зависимость не прямая, а квадратичная, то есть, если I увеличится стократно, то тепла выделится в десять тысяч раз больше. В этом и состоит опасность явления, приводящего порой к пожарам.

Провода накаляются докрасна (или добела), они передают эту энергию стенам, потолкам и другим предметам, которых касаются, и поджигают их. Если фаза в каком-то приборе касается нулевого проводника, возникает ток короткого замыкания источника, замкнутого на самого себя. Горючее основание электропроводки — страшный сон инспекторов пожарной охраны и причина многих штрафов, налагаемых на безответственных собственников зданий и помещений. И всему виной, конечно же, не законы Джоуля-Ленца и Ома, а пересохшая от старости изоляция, неаккуратно или безграмотно произведенный монтаж, повреждения механического характера или перегрузка проводки.

Однако и ток короткого замыкания, каким бы он ни был большим, также не бесконечен. На размеры бед, которые он может натворить, влияет продолжительность нагрева и параметры схемы электроснабжения.

Цепи переменного тока

Рассмотренные выше ситуации имели общий характер или касались цепей постоянного тока. В большинстве случаев электроснабжение и жилых, и промышленных объектов производится от сети переменного напряжения 220 или 380 Вольт. Неприятности с проводкой, рассчитанной на постоянный ток, чаще всего случаются в автомобилях.

Между этими двумя основными типами электропитания есть разница, и существенная. Дело в том, что прохождению переменного тока препятствуют дополнительные составляющие сопротивления, называемые реактивными и обусловленные волновой природой возникающих в них явлений. На переменный ток реагируют индуктивности и емкости. Ток короткого замыкания трансформатора ограничивается не только активным (или омическим, то есть таким, которое можно измерить карманным приборчиком-тестером) сопротивлением, но и его индуктивной составляющей. Второй тип нагрузки — емкостный. Относительно вектора активного тока векторы реактивных составляющих отклонены. Индуктивный ток отстает, а емкостный опережает его на 90 градусов.

Примером разницы поведения нагрузки, обладающей реактивной составляющей, может служить обычный динамик. Его некоторые любители громкой музыки перегружают до тех пор, пока диффузор магнитное поле не выбивает вперед. Катушка слетает с сердечника и тут же сгорает, потому что индуктивная составляющая ее напряжения уменьшается.

Виды КЗ

Ток короткого замыкания может возникать в разных цепях, подключенных к различным источникам постоянного или переменного тока. Проще всего дело обстоит с обычным плюсом, который вдруг соединился с минусом, минуя полезную нагрузку.

А вот с переменным током вариантов больше. Однофазный ток короткого замыкания возникает при соединении фазы с нейтралью или ее заземлении. В трехфазной сети может возникнуть нежелательный контакт между двумя фазами. Напряжение в 380 или более (при передаче энергии на большие расстояния по ЛЭП) вольт также может вызвать неприятные последствия, в том числе и дуговую вспышку в момент коммутации. Замкнуть может и все три (или четыре, вместе с нейтралью) провода одновременно, и ток трехфазного короткого замыкания будет течь по ним до тех пор, пока не сработает защитная автоматика.

Но и это еще не все. В роторах и статорах электрических машин (двигателей и генераторов) и трансформаторах порой случается такое неприятное явление, как межвитковое замыкание, при котором соседние петли провода образуют своеобразное кольцо. Этот замкнутый контур обладает крайне низким сопротивлением в сети переменного тока. Сила тока короткого замыкания в витках растет, это становится причиной нагрева всей машины. Собственно, если такая беда произошла, не следует ждать, пока оплавится вся изоляция и электромотор задымится. Обмотки машины нужно перематывать, для этого необходимо специальное оборудование. Это же касается и тех случаев, когда из-за «межвиткового» возник ток короткого замыкания трансформатора. Чем меньше обгорит изоляция, тем проще и дешевле будет перемотка.

Расчет величины тока при коротком замыкании

Каким бы ни было катастрофичным то или иное явление, для инженерной и прикладной науки важна его количественная оценка. Формула тока короткого замыкания очень похожа на закон Ома, просто к ней требуются некоторые пояснения. Итак:

I к.з.=Uph / (Zn + Zt),

I к.з. — величина тока короткого замыкания, А;

Uph — фазное напряжение, В;

Zn — полное (включая реактивную составляющую) сопротивление короткозамкнутой петли;

Zt — полное (включая реактивную составляющую) сопротивление трансформатора питания (силового), Ом.

Полные сопротивления определяются как гипотенуза прямоугольного треугольника, катеты которого представляют собой величины активного и реактивного (индуктивного) сопротивления. Это очень просто, нужно пользоваться теоремой Пифагора.

Несколько чаще, чем формула тока короткого замыкания, на практике используются экспериментально выведенные кривые. Они представляют собой зависимости величины I к.з. от длины проводника, сечения провода и мощности силового трансформатора. Графики представляют собой совокупность нисходящих по экспоненте линий, из которых остается лишь выбрать подходящую. Метод дает приблизительные результаты, но его точность вполне отвечает практическим потребностям инженеров по энергоснабжению.

Как проходит процесс

Кажется, что все происходит мгновенно. Что-то загудело, свет померк и тут же погас. На самом деле, как любое физическое явление, процесс можно мысленно растянуть, замедлить, проанализировать и разбить на фазы. До наступления аварийного момента цепь характеризуется установившимся значением тока, находящимся в пределах номинального режима. Внезапно полное сопротивление резко уменьшается до величины, близкой к нулю. Индуктивные составляющие (электродвигатели, дроссели и трансформаторы) нагрузки при этом как бы замедляют процесс роста тока. Таким образом, в первые микросекунды (до 0,01 сек) сила тока короткого замыкания источника напряжения остается практически неизменной и даже несколько снижается за счет начала переходного процесса. ЭДС его при этом постепенно достигает нулевого значения, затем проходит через него и устанавливается в каком-то стабилизированном значении, обеспечивающем протекание большого I к. з. Сам ток в момент переходного процесса представляет собой сумму из периодической и апериодической составляющих. Форма графика процесса анализируется, в результате чего можно определить постоянную величину времени, зависящую от угла наклона касательной к кривой разгона в точке ее перегиба (первой производной) и времени запаздывания, определяемого величиной реактивной (индуктивной) составляющей суммарного сопротивления.

Ударный ток КЗ

В технической литературе часто встречается термин «ударный ток короткого замыкания». Не следует пугаться этого понятия, оно вовсе не такое страшное и к поражению электричеством прямого отношения не имеет. Понятие это означает максимальное значение I к.з. в цепи переменного тока, достигающее своей величины обычно через полпериода после того, как возникла аварийная ситуация. При частоте 50 Гц период составляет 0,2 секунды, а его половина — соответственно 0,1 сек. В этот момент взаимодействие проводников, расположенных вблизи друг относительно друга, достигает наибольшей интенсивности. Ударный ток короткого замыкания определяется по формуле, которую в этой статье, предназначенной не для специалистов и даже не для студентов, приводить не имеет смысла. Она доступна в специальной литературе и учебниках. Само по себе это математическое выражение не представляет особой сложности, но требует довольно объемных комментариев, углубляющих читателя в теорию электроцепей.

Полезное КЗ

Казалось бы, очевидный факт состоит в том, что короткое замыкание — явление крайне скверное, неприятное и нежелательное. Оно может привести в лучшем случае к обесточиванию объекта, отключению аварийной защитной аппаратуры, а в худшем — к выгоранию проводки и даже пожару. Следовательно, все силы нужно сосредоточить на том, чтобы избежать этой напасти. Однако расчет токов короткого замыкания имеет вполне реальный и практический смысл. Изобретено немало технических средств, работающих в режиме высоких токовых значений. Примером может служить обычный сварочный аппарат, особенно дуговой, замыкающий в момент эксплуатации практически накоротко электрод с заземлением. Другой вопрос состоит в том, что режимы эти носят кратковременный характер, а мощность трансформатора позволяет выдерживать эти перегрузки. При сварке в точке касания окончания электрода проходят огромные токи (они измеряются в десятках ампер), в результате чего выделяется достаточно тепла для местного расплавления металла и создания прочного шва.

Методы защиты

В первые же годы бурного развития электротехники, когда человечество еще отважно экспериментировало, внедряя гальванические приборы, изобретало различные виды генераторов, двигателей и освещения, возникла проблема защиты этих устройств от перегрузок и токов короткого замыкания. Самое простое ее решение состояло в последовательной с нагрузкой установке плавких элементов, которые разрушались под воздействием резистивного тепла, в случае если ток превышал установленное значение. Такие предохранители служат людям и сегодня, их главные достоинства состоят в простоте, надежности и дешевизне. Но есть у них и недостатки. Сама простота «пробки» (так назвали держатели плавких ставок за их специфическую форму) провоцирует пользователей после ее перегорания не мудрствовать лукаво, а заменять вышедшие из строя элементы первыми попавшимися под руку проволочками, скрепками, а то и гвоздями. Стоит ли упоминать о том, что такая защита от токов короткого замыкания не выполняет своей благородной функции?

На промышленных предприятиях для обесточивания перегруженных цепей автоматические выключатели начали использовать раньше, чем в квартирных щитках, но в последние десятилетия «пробки» были в основном заменены ими. «Автоматы» намного удобнее, их можно не менять, а включить, устранив причину КЗ и дождавшись, когда тепловые элементы остынут. Контакты у них иногда подгорают, в этом случае их лучше заменить и не пытаться почистить или починить. Более сложные дифференциальные автоматы при высокой стоимости не служат дольше обычных, но функционально их нагрузка шире, они отключают напряжение в случае минимальной утечки тока «на сторону», например при поражении человека током.

В обыденной же жизни экспериментировать с коротким замыканием не рекомендуется.

Короткое замыкание и как его предотвратить

Основная причина пожаров, связанных с нарушением правил устройства и эксплуатации электрооборудования — это короткое замыкание.

Оно возникает из-за нарушения изоляции в электропроводах и кабелях, вызываемое перенапряжением, износом изоляции и механическими повреждениями. Опасность короткого замыкания заключается в увеличении силы тока на сотни тысяч ампер, из-за чего происходит выделение большого количества тепла в проводниках за очень короткий промежуток времени, что в свою очередь, приводит к резкому повышению температуры и воспламенению изоляции.

Чтобы избежать неприятных последствий короткого замыкания, необходимо помнить несколько простых правил:

— Категорически запрещается соединять провода в виде скрутки, так как плотность контактов проводников такого соединения быстро ослабевает и уменьшается площадь их взаимодействия, из-за этого возможно возникновение электрической дуги и искрения, а в последствие и короткого замыкания.

— При проведении скрытой электропроводки, например, за подвесными потолками, в зависимости от степени горючести используемого материала, необходимо использовать кабель не распространяющий горение, или помещать его в стальные трубы с определенной толщиной стенки, которая не прожжется в результате возникновения короткого замыкания.

— Необходимо использовать электропредохранитель, с соответствующей для вашего потребления мощностью. Если предохранитель часто отключается, то возможно где-то есть неполадки в соединениях проводов или вы потребляете большее количество электроэнергии, чем рассчитан ваш предохранитель. В этом случае надо вызвать специалиста, чтобы избежать печальных последствий замыкания сети.

Будьте внимательны и осторожны при обращении с электроприборами! Берегите себя и своих близких!

 

Информация с сайта http://78.mchs.gov.ru/

Отдел надзорной деятельности Центрального района

УНД ГУ МЧС России по городу Санкт‑Петербургу

Как определить короткое замыкание

Причины появления

При образовании контакта между двумя потоками зарядов возникает огромная сила тока, которой характеризуется КЗ. Так как это резкое возрастание энергии происходит из-за появления ударного электрического импульса, то, согласно закону Джоуля — Ленца, появившаяся мощность имеет высокую степень выделения тепла.

Причина такого явления заключается в нарушении изоляционных свойств проводников. Это может произойти из-за естественного износа или возникновения аварийных ситуаций в работе электрооборудования. На практике это обозначает непосредственный контакт между нулевым и фазовым проводом.

Но провода необязательно должны соприкоснуться между собой. Мгновенный рост силы тока также возникает, если между ними появляется проводящее тело или среда с ничтожно малым сопротивлением, например, проводящий предмет, вода или влажный воздух. Такую ситуацию называют пробоем. Можно выделить следующие причины возникновения КЗ:

Перенапряжение. При превышении физических параметров электролинии, когда по проводнику проходит большая сила тока, его сопротивление оказывается для неё большим, поэтому происходит интенсивное выделение тепла. Проводник греется, передавая тепло изоляционному слою, который растрескивается и разрушается. Появление высокого напряжения может быть вызвано как природными явлениями (ударом молнии), так и ошибками в работе электроприборов или источников энергии.
Естественное старение изоляции. Любая изоляция имеет свой ресурс, который сокращается при нахождении в агрессивной окружающей среде, например, при перепадах температуры.
Механическое повреждение: перетирание, повреждение при выполнении строительных работ, результат действия грызунов.
Ошибка монтажа

Неосторожное обращение с токоведущими частями оборудования, нарушения в укладке кабеля (сгибание под острым углом, пересечение проводов).
Неправильная коммутация электрооборудования или кабелей. Происходит из-за действий человека

Это может быть связано с неудачным ремонтом, ошибкой соединения проводов или электролиний, использованием сломанных устройств.

Чем КЗ отличается от перегрузки

Если фазу и ноль электрической сети соединить под напряжением друг с другом не через потребитель, а напрямую, то возникнет короткое замыкание, сокращенно КЗ. Коротким замыканием называется соединение проводников отдельных фаз между собой или с землей через относительно малое сопротивление, принимаемое равным нулю при глухом металлическом коротком замыкании.

Никакая сеть не предназначена для длительной работы в таком режиме. Однако данный аварийный режим иногда возникает. Так, короткое замыкание может случиться из-за нарушения изоляции электропроводки или из-за случайного замыкания разноименных проводников проводящими частями электрооборудования. Нормальная работа электрической сети будет нарушена. Чтобы это нежелательное явление предотвратить, электрики используют клеммники либо просто изолируют соединения.

Проблема режима КЗ заключается в том, что в момент его возникновения в сети многократно увеличивается ток (до 20 раз превышает номинал), что приводит к выделению огромного количества джоулева тепла (до 400 раз превышает норму), поскольку количество выделяемой теплоты пропорционально квадрату тока и сопротивлению потребителя.

Теперь представьте: сопротивление потребителя здесь — доли ома проводки, а ток, как известно, тем выше, чем меньше сопротивление. В итоге, если мгновенно не сработает защитное устройство, произойдет чрезмерный перегрев проводки, провода расплавятся, изоляция воспламенится, и может случиться пожар в помещении. В соседних помещениях, питаемых этой же сетью, упадет напряжение, и некоторые электроприборы могут выйти из строя.

Типичный вид короткого замыкания для жилых квартир — однофазное короткое замыкание, когда фаза смыкается с нулем. Для сетей трехфазных, например в цеху или в гараже, возможно трехфазное или двухфазное короткое замыкание (две фазы между собой, три фазы между собой, или несколько фаз на ноль). Для трехфазного оборудования, такого как асинхронный двигатель или трехфазный трансформатор, характерно межвитковое замыкание, когда витки замыкаются накоротко внутри обмотки статора или внутри обмотки трансформатора, шунтируя остальные рабочие витки и выводя таким образом прибор из строя.

Или замыкание может случиться через проводящий корпус прибора. Вообще проводящие корпуса следует заземлять, дабы защитить персонал от случайного поражения током, а провода в квартирах использовать те, что в негорючей изоляции. Есть еще один вид аварийного режима нагрузки электрической сети, связанный с превышением нормального тока.

Это так называемая перегрузка. Перегрузки иногда возникают в квартирах, в домах, на предприятиях. Это опасный режим, порой более опасный, чем короткое замыкание. Ведь короткое замыкание в квартире может быть на корню остановлено мгновенно сработавшим автоматическим выключателем в щитке. А вот токовая перегрузка — случай более хитрый.

Выключатели для защиты от короткого замыкания.

Представьте себе, что в одну единственную розетку вы решили понавтыкать множество электроприборов через тройник да через удлинители. Что нежелательного может в этом случае произойти? Если жила проводки, подведенный к розетке, не рассчитана на ток более 16 ампер, то при включении в такую розетку нагрузки более 3500 ватт начнется перегрев электропроводки чреватый пожаром.

Вообще тепловое воздействие на изоляцию проводов резко снижает ее механические и диэлектрические свойства. Например, если проводимость электрокартона (как изоляционного материала) при 20°С принять за единицу, то при температурах 30, 40 и 50°С она увеличится в 4, 13 и 37 раз соответственно.

И тепловое старение изоляции наиболее часто возникает именно из-за перегрузки электросетей токами, превышающими длительно допустимые для данного вида и сечений проводников. Также нельзя в розетку, на которой указано 250 В 10 А, включать потребителей более чем на 2500 Вт, ибо может начаться перегрев контактов, ведущий к их ускоренному окислению. Для защиты от перегрузок в квартире, а также для мгновенного купирования режима КЗ, используйте автоматические выключатели.

Какие бывают виды

Короткое замыкание. Каждый слышал это словосочетание. Многие видели надпись «Не закорачивать!» Часто, когда ломается какой-нибудь электроприбор, говорят: «Коротнуло!» И несмотря на негативный оттенок этих слов, профессионалы знают, что короткое замыкание – не печальный приговор. Иногда с коротким замыканием (КЗ) бороться бессмысленно, а порой и принципиально невозможно. В этой статье будут даны ответы на самые важные вопросы: что такое короткое замыкание и какие виды КЗ встречаются в технике.

Начнем рассматривать эти вопросы под необычным углом – узнаем, в каких случаях короткие замыкания неизбежны и где они не играют роль повреждений. Возьмем за оба конца обыкновенный металлический провод. Соединим концы вместе. Провод замкнулся накоротко – произошло КЗ. Но так как в цепи отсутствуют источники электрической энергии и нагрузка, такое короткое замыкание никакого вреда не несет. В некоторых областях электротехники КЗ, которое мы рассмотрели, играет на руку, например, в электрических аппаратах и электрических машинах.

Взглянем на однофазное реле или пускатель, в конструкции которых есть магнитная система с подвижными частями – электромагнит, притягивающий якорь. Из-за постоянно меняющейся полярности тока, текущего в обмотках электромагнита, его магнитный поток периодически становится равен нулю, что вызывает дребезжание якоря, появляются вибрации и характерное, знакомое всем электрикам гудение. Чтобы избавиться от этого явления, на торец сердечника электромагнита или якоря прикрепляют короткозамкнутый виток – кольцо или прямоугольник из меди или алюминия.

Из-за явления электромагнитной индукции в витке создается ток, создающий свой магнитный поток, компенсирующий пропадание основного магнитного потока, создаваемого электромагнитом, что приводит к уменьшению или исчезновению вибраций, разрушающих конструкцию.

Так же на руку играет короткое замыкание и в роторе асинхронного электродвигателя. Благодаря взаимодействию магнитного поля, создаваемого обмотками статора, с короткозамкнутым ротором, в роторе по уже упомянутому закону появляются свои токи, создающие свое поле, что приводит ротор во вращение

Конечно, важно грамотное проектирование электродвигателя или электрического аппарата, чтобы токи, протекающие в короткозамкнутых элементах, не приводили к перегреву и порче изоляции основных обмоток

Возгорание розетки

Подобным образом понятие «короткое замыкание» используется применительно к трансформаторам. Люди, так или иначе связанные с энергетикой, знают, что одна из важнейших характеристик трансформатора – это напряжение короткого замыкания, UКЗ, измеряемое в процентах. Возьмем трансформатор. Одну из его обмоток, скажем, низшего напряжения (НН) закоротим амперметром, сопротивление которого, как известно, принимается равным нулю. Обмотку высшего напряжения (ВН) подключаем к источнику напряжения. Повышаем напряжение на обмотке ВН до тех пор, пока ток в обмотке НН не станет равным номинальному, фиксируем это напряжение.

Делим его на номинальное напряжение высшей стороны, умножаем на 100%, получаем UКЗ. Эта величина характеризует потери мощности в трансформаторе и его сопротивление, от которого зависит ток короткого замыкания, ведущий к повреждениям. Поговорим наконец о коротких замыканиях, несущих негативные последствия. Такие короткие замыкания появляются, когда ток от источника питания протекает не через нагрузку, а только через провода, обладающие ничтожно маленьким сопротивлением. Например, трехфазный кабель питается от трансформатора, и одним неосторожным движением ковша экскаватора происходит его повреждение – две фазы закорачиваются через ковш. Такое КЗ называют двухфазным. Аналогично по количеству замкнутых фаз называют другие КЗ.

Однофазное замыкание на землю в сетях с изолированной нейтралью не является коротким, но может представлять угрозу жизни живых существ.  Металлическим называют КЗ, в котором переходное сопротивление равно нулю – например, при болтовом или сварочном соединении. Токи КЗ в зависимости от напряжения и вида повреждения могут достигать тысяч и сотен тысяч ампер, приводить к пожарам и колоссальным электродинамическим усилиям, «выворачивающим» шины и провода. Защита от КЗ может осуществляться автоматическими выключателями или предохранителями, а в высоковольтных сетях – средствами релейной защиты и автоматики.

Защита блока питания от короткого замыкания.

Что же такое короткое замыкание?

Пример: Локомотив должен доставить груз, допустим из города Нижний Новгород в такой  мегаполис  как Москва. Путь состава должен быть длинным.  Локомотив, таща за собой 50 вагонов угля, набирает большую скорость. Но вдруг, в городе Владимир диспетчер совершает роковую ошибку, переключив стрелку на путь, где находится другой состав — аварии не миновать.

Состав набравший большую скорость быстро не остановить. Наглядный пример может показаться примитивным, но хочется показать принцип лежащий в основе – это сила, мощь, использованная не по назначению, несущая разрушение. Путь следования локомотива с множеством вагонов оказался коротким, не завершенным, не достиг цели.

Защита от токов короткого замыкания

Как мы выяснили, токи КЗ весьма опасны, прежде всего с точки зрения пожарной безопасности. Поэтому необходимо построить защиту от токов короткого замыкания, то есть установить в щите автоматические выключатели. Автоматические предохранители устроены так, что в случае короткого замыкания рост тока КЗ приводит к срабатыванию электромагнитного расцепителя мгновенного действия, который разъединяет электрическую цепь без ущерба для себя.

Для того, чтобы после устранения короткого замыкания снова включить электричество, необходимо просто нажать на белую кнопку (красная служит для выключения) или перекинуть вверх опустившийся при срабатывании предохранителя рычажок.

Правила монтажа электропроводки предусматривают расчет нагрузки и токов, идущих через автоматы защиты. Понятно, что предохранитель должен срабатывать при значениях тока, выбранных с солидным запасом. Иначе случайные небольшие колебания напряжения в сети (а следовательно, и тока) будут приводить к постоянному ложному срабатыванию защиты. С другой стороны, запас не должен быть и слишком велик, чтобы действия тока не причинило вреда сети раньше, чем произойдет отсечка.

Возгорание изоляции кабеля.

Автоматический предохранитель защищает внутреннюю и внешнюю сеть

Заметим, что автоматические предохранители, установленные в начале каждой домовой линии (рабочей группы), защищают от короткого замыкания не только домовую сеть, но и наружную. В самом деле, если бы их не было, то аварийное короткое замыкание привело бы к выходу из строя трансформаторной подстанции, а вернее, электрического силового щита более высокого уровня, так что электричества лишилось бы значительное количество пользователей, да и без вызова аварийной службы было бы не обойтись. А при наличии «автомата» достаточно включить его после срабатывания (удалив, конечно, причину короткого замыкания).

Становится понятна и необходимость нескольких линий в доме: если одна линия вылетела, в запасе есть другие. Кстати, отсюда вывод: удобно, если от каждой рабочей группы питается лампочка аварийного освещения в районе счетчика или аварийная розетка, в которую можно включить переносную лампу.

Как образуется короткое замыкание

Как мы помним из учебника физики за 8 класс, закон Ома для участка цепи определяется по формуле:

где

I – сила тока в цепи, А

U – напряжение, В

R – сопротивление, Ом

Давайте рассмотрим вот такую схему

Если мы подключим настольную лампу EL к источнику тока Bat и замкнем ключ SA, то вольфрамовая нить лампы начнет разогреваться под тепловым воздействием тока. В этом случае значительная часть электрической энергии преобразуется в световую и тепловую.

А теперь покончим с лирическими отступлениями и замкнем два провода, которые идут на лампочку, через толстый провод AВ

Что будет дальше, если мы замкнем контакты ключа SA?

В результате ток пойдет по укороченному пути, минуя нагрузку. Короткий путь в данном случае и есть провод AB. Сопротивление провода АВ близко к нулю. В результате наша схема преобразуется в делитель тока. Согласно правилу делителя тока, если нагрузки соединены параллельно, то через нагрузку с меньшим сопротивлением побежит большая сила тока, а через нагрузку с большим значением сопротивления – меньшая сила тока. Так как провод АВ обладает почти нулевым сопротивлением, то через него потечет большая сила тока, согласно опять же закону Ома:

Как я уже сказал, в режиме КЗ сила тока достигает критических значений, превышающих допустимые для данной цепи.

Меры, исключающие короткое замыкание

Еще на заре развития электротехники появились плавкие предохранители. Принцип действия подобной защиты очень прост: под влиянием теплового действия тока предохранитель разрушается, тем самым размыкая цепь. Предохранители наиболее часто используются в бытовых электросетях и бытовых электроприборах, электрическом оборудовании транспортных средств и промышленном электрооборудовании до 1000 В. Встречаются они и в цепях с высоковольтным оборудованием.

Вот такие предохранители используются в цепях с малыми токами

вот такие плавкие предохранители вы можете увидеть в автомобилях

А вот эти большие предохранители используются в промышленности, и они уже рассчитаны на очень большие значения токов

Более сложную конструкцию имеют автоматические выключатели, оснащенные электромагнитными и/или тепловыми датчиками. Ниже на фото однофазный автоматический выключатель, а справа – трехфазный

Их принцип действия основан на размыкании цепи при превышении допустимых значений силы тока.

В быту мы чаще всего сталкиваемся со следующими устройствами защиты электросети:

  • Плавкие предохранители (применяются в том числе в бытовых электроприборах).
  • Автоматические выключатели.
  • Стабилизаторы напряжения.
  • Устройства дифференциального тока.

Все вышеперечисленное защитное оборудование относится к устройствам вторичной защиты, действующим по инерционному принципу. На вводе бытовых электросетей наиболее часто устанавливаются автоматические защитные устройства, действующие по адаптивному принципу. Такие устройства можно увидеть возле счетчиков электроэнергии квартир, коттеджей, офисов.

В высоковольтных сетях защита чаще обеспечивается:

  • Устройствами релейной защиты и другим отключающим оборудованием.
  • Понижающими трансформаторами.
  • Распараллеливанием цепей.
  • Токоограничивающими реакторами.

Большинства коротких замыканий можно избежать, если устранить основные причины их возникновения: своевременно ремонтировать или заменять изношенное оборудование, исключить вредные воздействия человека. Не допускать неправильных действий при монтажных и ремонтных работах, соблюдать СНИПы и правила техники безопасности.

Вот еще одно яркое объяснение, что такое короткое замыкание

В нормальном режиме работы ток в проводке между фазным и нулевым проводами протекает через нагрузку, которая этот ток ограничивает на безопасном для проводки уровне. При разрушении изоляции ток протекает, минуя нагрузку, сразу между проводами. Такой контакт, называется коротким, поскольку происходит помимо электроприбора.

Вспомним закон Ома: I = U/R, что словами, обычно, произносится так: «Ток в цепи прямо пропорционален напряжению, и обратно пропорционален СОПРОТИВЛЕНИЮ»

Именно на СОПРОТИВЛЕНИЕ здесь и стоит обратить внимание

Сопротивление ТПЖ электропроводки, как правило, невелико, поэтому им можно пренебречь, считать его равным нулю. Согласно законам математики деление на ноль невозможно, а результат будет стремиться к бесконечности. В случае короткого замыкания к этой самой бесконечности будет стремиться ток в цепи.

Конечно, это не совсем так, провода имеют какое-то конечное сопротивление, поэтому до бесконечности ток, конечно же, не дойдет, но будет достаточной силы, чтобы произвести разрушительное действие, достаточно мощный взрыв. Возникает вольтова дуга, температура которой достигает 5000 градусов по Цельсию.

Меры профилактики по предотвращению КЗ

Предотвратить возникновение коротких замыканий в значительной степени помогают меры предосторожности и профилактические мероприятия

Наиболее важными из них являются следующие:

Перед тем как найти короткое замыкание в проводке, следует обращать внимание на заметное искрение или треск в розетке и выключателях, сопровождающиеся запахом горелой пластмассы. Именно эти факторы чаще всего приводят к аварийному режиму

В таких случаях нужно обязательно заменить неисправные установочные изделия.
Перед прокладкой новых проводов нужно заранее рассчитать мощность потребителей, которые будут использованы в данной сети. Правильно выбранное сечение предохраняет кабели от излишних перегрузок. В процессе монтажа проводка не должна быть перекручена. Кабели укладываются параллельно, а между ними оставляется свободное пространство.
При выполнении ремонтных работ, связанных со сверлением стен, необходимо заранее уточнить места прокладки кабельных линий.
Установка средств автоматической защиты позволит избежать негативных последствий, за счет отключения линии в момент КЗ.
Не реже 2-3 раз в год проводить плановые осмотры, выключателей, розеток, распределительных коробок, откуда расходятся провода и мощного оборудования. Проводку с алюминиевыми жилами по возможности лучше устраняем и меняем, поскольку этот материал при нагревании увеличивает сопротивление цепи, вызывая увеличенный нагрев, замыкание и расплавление кабельных линий.
При поиске короткого замыкания все действия с проводкой и электрическими приборами следует выполнять при строгом соблюдении техники безопасности. Выполняя рекомендации специалистов и точно соблюдая порядок действий, вполне возможно не только самостоятельно обнаружить аварию, но также исправить все ее последствия.

Неполадки в электросети являются частой причиной поломки электроприборов. Одно из распространенных негативных явлений – короткое замыкание. Оно опасно не только для техники, но и для самого дома, так как может привести к возгоранию. Бороться с КЗ можно с помощью предохранителей и автоматов защиты в активной цепи. Но в случае обесточенного провода или прибора поиск замыкания усложняется. Есть разные способы обнаружения и устранения дефекта сети.

Последствия и защита

Самая большая неприятность, которая может возникнуть во время КЗ, — это возникновение пожара. Образовавшийся избыток тепла вызывает возгорание изоляции и близко расположенных горючих веществ. Кроме того, при пробое возникает электрическая дуга, представляющая собой открытый источник огня. При этом она может воспламенить рядом находящиеся предметы.

Если в зону действия замыкания попадает человек, то он подвергается удару большой силы тока. Это приводит к выделению организмом тепла и последующему сгоранию тела.

К менее значимым последствиям можно отнести возникновение электрического поля большой величины и электромагнитного удара, которые негативно воздействуют на электронные узлы и блоки радиоаппаратуры, а также электродвигатели. Появление КЗ приводит к перекосу фаз в электросети, что вызывает в ней всплеск напряжения, а это ведёт к поломкам блоков питания электрических устройств, подключённых к ней.

Для предотвращения последствий при конструировании электрических систем используются специальные устройства и мероприятия по защите. К таким мерам относят:

  • периодическое испытание изоляции оборудования и линий электропередач;
  • использование техники или электропроводки с повышенным классом защиты в потенциально опасных местах, например, применение высококачественных диэлектриков или не поддерживающей горение двойной изоляции;
  • установка предохранителей, перегорающих при достижении тока опасной величины;
  • использование громозащиты, автоматических и дифференциальных выключателей, устройств контроля напряжения.

Преднамеренное использование

Непреднамеренное возникновение КЗ несёт в себе опасности и неприятности, но контролируя его, можно извлечь выгоду. Благодаря этому появилась электродуговая сварка. Одна обмотка трансформатора подключается к свариваемой детали, образуя с ней постоянный контакт, а вторая соединяется с электродом. При его прикосновении к детали возникает КЗ с образованием дуги и выделением тепла. Именно эта дуга и применяется для расплавления металлов. Но если электрод «прилипнет» к металлу, то вся используемая мощность начнёт выделяться на преобразователе, что приведёт к его межвитковому замыканию.

Другое применение явления используется в короткозамыкателях — устройствах, оборудованных автоматическими выключателями. При возникновении необходимости быстро отключить линию электропередачи происходит её замыкание, на которую реагирует электромеханическое устройство, отключающее участок.

Индукционные виброметры и сейсмоприёмники в основе своей работы используют КЗ, что позволяет демпфировать механические колебания. Часто режим пробоя используется при соединении усилительных каскадов в электронике, например, каскодных усилителей. Цепи питания электронных плат тоже работают в режиме КЗ для переменного тока. На их линиях питания устанавливаются шунтирующие блокировочные конденсаторы. Предназначены они для снижения самовозбуждения усилительных каскадов, помех и сбоев кодов в цифровых устройствах.

Подводя итоги, можно отметить важные аспекты электрических процессов. Для того чтобы возник ток, должна появиться разность потенциалов, а затем физическое их соединение. Тогда заряд начнёт передаваться из одной точки в другую, встречая на своём пути сопротивление. На нём будет выделяться энергия, которая используется для каких-либо процессов. Образованная мощность характеризуется полезной работой. Но если на своём пути ток не встретит препятствия, то при соединении двух потенциалов вся мощность выделится на этом пути. В результате она преобразуется в тепловую энергию и частично в световую.

Это и есть принцип возникновения КЗ, а также явлений, сопровождающих его, — чрезмерного нагрева и световой вспышки. При возникновении такого режима ток в цепи намного превышает номинальные значения (из-за отсутствия сопротивления), что и приводит к выходу из строя источника энергии и её приёмника в электрической цепи.

Проверка электропроводки

Если с предохранителями все очень просто и вместо сгоревшего достаточно установить новый, то с электропроводкой все намного сложнее. Основным признаком, по которому определяется короткое замыкание в автомобиле, является повторно сгоревший предохранитель, который был только что заменен. Поэтому причину необходимо искать в проводке, используя для этого метод исключения.

В этом случае разные потребители поочередно отключаются от проверяемой линии. Одновременно проверяется целостность проводов. Часто место аварии определяется визуально, по следам подгоревшей и оплавившейся изоляции. Однако, некоторые провода спрятаны слишком далеко, например, под панелью приборов, поэтому приоритет в проверке отдается потребителям.

Если в проверяемой линии автомобильной проводки присутствует короткое замыкание, то в момент включения лампочка загорится ярким светом на полную мощность. Когда она подключена к исправной линии, то свечение будет тусклым, поскольку ток на этом участке имеет номинальное значение. Так методом исключения можно поочередно узнать состояние всех потребителей. Когда обнаруживается неисправный проверяемый потребитель, можно попытаться его отремонтировать. Однако, специалисты рекомендуют не рисковать и установить новую деталь. С замыканием шутить не следует, поскольку в другом случае при несчастливом стечении обстоятельств, может сгореть вся проводка и даже сам автомобиль.

Иногда может возникнуть ситуация, когда все потребители уже проверены, а лампочка все равно светит на полную мощность, то есть, причина все еще не найдена. Это указывает на наличие короткого замыкания в самой проводке, а не в потребителях. В этом случае проверка существенно осложняется, поскольку для доступа ко всем линиям придется демонтировать элементы салона и другие детали.

Тем не менее, тестером, если он есть в наличии, можно проверить участки, находящиеся в свободном доступе и существенно сократить поиски. Эти задачи уже более сложные и требуют вмешательства квалифицированных автомобильных электриков. Кроме того, водителю потребуются знания и навыки разборки и сборки различных элементов авто, правила пользования обычным и специальным инструментом.

Как найти короткое замыкание

Что такое короткое замыкание, его виды и причины возникновения

Что такое ток короткого замыкания

Причины возникновения короткого замыкания

Как рассчитать ток короткого замыкания

Режим короткого замыкания

Короткое замыкание, методы его поиска

Рассмотрим случай, когда у вас перегорает предохранитель. Существует несколько видов короткого замыкания, в первом случае у вас предохранитель сгорает моментально, сразу после его установки в блок предохранителей или в течении короткого промежутка, в считанные моменты после установки, но не моментально. Это означает, что в первом случае, проводник по которому идёт «плюс» имеет прямой непосредственный контакт с «массой» корпусом автомобиля, такой вариант немного проще в поиске неисправности. Всегда труднее выявлять причину неисправности, когда она слабая или имеет спонтанный характер, склонность то к появлению, то к исчезновению.

При поиске короткого замыкания, самое главное определить на каком участке цепи происходит замыкание. Рекомендую вам при поиске вооружится контрольной лампочкой, длинным куском провода, книгой или электрической схемой с электрооборудованием вашего автомобиля. Если у вас перегорает предохранитель, значит короткое замыкание расположено на участке цепи от блока предохранителя к тому устройству которое запитывается от этого предохранителя.

Как правило само короткое замыкание происходит или на пути к этому устройству в проводке, в местах соединения проводки с этим устройством или в самом устройстве. Таким образом определив номер предохранителя, посмотрите, за какие цепи отвечает этот предохранитель и по возможности отключите все потребители электроэнергии в местах где проводка подходит к ним.

Я написал «по возможности», потому что не все потребители электроэнергии легко доступны, а многие расположены в местах, где эту процедуру сможет произвести только специалист или человек уже сталкивавшийся с этим. Например: если вы определите что у вас замыкание в цепи указателей поворотов или аварийной сигнализации автомобиля ВАЗ 2105, нужно будет отсоединить штекерную колодку от выключателя аварийной сигнализации, которая расположена справа на рулевой колонке, если при её отключении и установлении предохранителя, короткое замыкание пропадает, нужно идти дальше по проводке, а реле указателей поворотников и аварийной сигнализации, расположено под приборной панелью, снять которую в одиночку я вам не советую, если у вас такого опыта не было.

Выше я советовал при поиске воспользоваться контрольной лампочкой и куском провода, сейчас поясню зачем. Как мы определили короткое замыкание — это когда проводка или какая то часть устройства, соединяется или имеет контакт с массой автомобиля, делаем цепь соединяя длинный провод с «плюсом» аккумулятора и контрольной лампочкой, впрочем это может быть просто контрольная лампочка с длинной одной частью. При поиске короткого замыкания аккумулятор у вас должен быть отсоединён, но в данном случае, вам нужно будет полностью всё отсоединить от «плюса», а масса «минус» должна быть подсоединена к автомобилю.

Подсоединяем второй вывод нашей контрольной лампы к блоку предохранителей, соединяем его с местом, откуда выходит этот самый образовавшийся минус, так как при вставке предохранителя он у нас сразу перегорает, значит на одном выходе образуется минус, а на второй как и положено подходит положительное напряжение. При подсоединении лампочки на этот вывод, она у нас в случае короткого замыкания, сразу ярко загорится, теперь будем по схеме отключать все потребители электроэнергии, которые запитаны на этот предохранитель. И будем наблюдать за поведением лампочки, за тем как она горит, как только мы отсоединим устройство, дающее короткое замыкание, лампочка должна или погаснуть или тускней светить, что будет соответствовать тому, что мы нашли причину короткого замыкания.

Профилактика КЗ

Выполнить профилактические действия безопаснее, надежнее и дешевле, чем восстанавливать проводку после КЗ. Периодически нужно проверять розетки. Если они начинают искрить, нужно их ремонтировать или менять.

Если производилась частичная замена проводки, следует проверять надежность мест соединения, целостность изоляционного слоя.

Раз в несколько месяцев следует проверять источники света, осветительную сеть и силовые провода. Короткое замыкание может возникать со временем. Выявить его можно по изменению цвета устройств или их плавлению. В квартире обязательно должны стоять автоматические выключатели. На мощные электроприборы ставятся отдельные средства защиты, которые должны сработать при аварийной ситуации.

При самостоятельном проведении монтажа электропроводки важно правильно рассчитывать сечение кабеля. Если оно не способно выдержать мощность всех подключаемых приборов, будет происходить перегрузка, приводящая к короткому замыканию

Кабели не должны укладываться тесно друг с другом – это может привести к повреждению защитного слоя. Также при соединении надо правильно выбрать способ создания контакта и приобрести заранее необходимое оборудование. Нельзя соединять провода методом скрутки.

Если надо сверлить стену, следует проверить место самодельным металлоискателем или изучить схему электропроводки. Таким образом можно обнаружить кабель скрытой проводки, который мастер мог бы случайно повредить.

Опасность и последствия

Чтобы понять, какую опасность представляет КЗ, достаточно узнать о возможных последствиях короткого замыкания. Для этого перейдем к краткому перечню, составленному по статистическим данным Ростехнадзора:

  • Возникновение возгорания в месте механического соприкосновения неизолированных элементов оборудования или электрической сети часто становится причиной пожара.
  • Понижение уровня напряжения электрического тока в зоне замыкания вызовет сбой в работе электрооборудования. О последствиях пониженного напряжения можно подробно узнать в одной из публикаций на нашем сайте.
  • Как видно из приведенной выше таблицы 1, на долю симметричных замыканий (К(З)) приходится не более 5%, это означает, что во всех остальных случаях придется иметь дело с сетевой асимметрией, более известной под названием «перекос фаз». Последствия такого режима мы уже рассматривали в более ранней публикации.
  • Возникновение различных системных аварий, вызывающих отключение потребителей энергосистемы до устранения короткого замыкания.

Как найти короткое замыкание в проводке

Как правило, поиск замыкания происходит уже после того, как выбило пробки или автоматический выключатель.

Тут есть несколько вариантов:

  • внешний осмотр;
  • использование специальных приборов;
  • исключением;
  • по звуку;
  • по запаху.

Внешний осмотр при коротком замыкании

Если вы обнаружили, что повреждена изоляция или соприкосновение двух оголенных жил – можете считать, что причина найдена.

Обычно, такие повреждения можно найти в распределительных коробках, выключателях или розетках, где соединяются провода.

Заметили обгорелую оболочку – это и есть неисправность.

Как найти короткое замыкания, используя приборы

Использовать для этого лучше мегаомметр или мультимерт. Они быстро проверят сопротивление в цепи.

Подключите один провод прибора к фазе, а другой к заземлению (к нулю).

Если прибор показывает ноль – проводка в норме. Все, что выше нуля свидетельствует о соприкосновении контактов.

Стоит учесть, что мультиметр имеет маленькое сопротивление, поэтому определить короткое замыкание с его помощью не всегда возможно.

Как найти замыкание методом исключения

Тут все просто, но способ эффективен в случае вины электроприбора.

Когда у вас выбило выключатель, выключите всю технику от электричества.

Затем включите автомат и начинайте подключать каждый из приборов.

Как найти короткое замыкание по звуку и запаху

При замыкании контактов можно услышать потрескивание. Главное иметь хороший слух. По запаху гари пластмассы и легкого дымка вы легко найдете обрыв проводки в доме.

4 основных причины короткого замыкания

Короткое замыкание возникает, когда электричество отклоняется от намеченного пути и завершает свой путь более коротким путем — маршрутом с меньшим сопротивлением. Это происходит, когда подача электричества прерывается из-за неисправности установленной проводки. Если короткое замыкание в вашей электрической системе не будет обнаружено и своевременно устранено, оно может вызвать искры, дым, пожар или поражение электрическим током.

Прежде чем мы перейдем к распространенным причинам короткого замыкания в доме, краткий урок естествознания поможет вам лучше понять, как работают электрические цепи.

Провода для электрических цепей

Для замыкания цепи требуются два провода: горячий провод (несущий отрицательный электрический заряд) и нейтральный провод (несущий положительный электрический заряд). Если в системе есть неисправность и ток выходит из любого из этих проводов, то в дело вступает заземляющий провод. Заземляющий провод проходит параллельно горячему и нейтральному проводу и, по сути, является резервным проводом, который направляет ток обратно на землю, если он отклоняется от предполагаемого пути.Если в вашем доме нет системы заземления, существует повышенный риск возгорания.

Короткое замыкание и замыкание на землю

Короткое замыкание и замыкание на землю — это термины, относящиеся к электрическим коротким замыканиям. Когда электрики используют термин «короткое замыкание», они обычно имеют в виду ситуацию, когда горячий провод касается нейтрального провода. С другой стороны, «замыкание на землю» происходит, когда горячий провод касается заземленной части вашей электрической системы, такой как металлическая настенная коробка или прибор.В обоих случаях большой ток выходит за пределы предполагаемого пути.

Распространенные причины короткого замыкания

1. Плохие соединения

Если крепления проводов ослабнут и провисают, два провода могут столкнуться друг с другом и вызвать короткое замыкание. Если вы заметили, что горячий провод и нейтральный провод соприкасаются, не затягивайте провода самостоятельно; вы можете быть сильно шокированы. Вызовите электрика, чтобы решить проблему за вас.

2. Старение и устаревшая проводка

Провода и изоляция проводов со временем изнашиваются.Типичный срок службы электрической системы составляет 30–40 лет. Если ваша система старше этой, она может быть небезопасной.

Если ваш дом был построен в 1970-х годах или ранее, в нем все еще может быть алюминиевая проводка. Алюминий быстрее изнашивается и легче перегревается, чем медь, что подвергает ваш дом большему риску возгорания. Кроме того, старые системы электропроводки не были рассчитаны на то, чтобы справляться с количеством электроэнергии, которое требуется современным домам.

Если электрическая система вашего дома стареет, возможно, пришло время обновить ее.Вызовите доверенного электрика, который проведет осмотр и порекомендует лучший способ действий.

3. Неисправность электропроводки прибора

Короткие замыкания могут возникать в приборах из-за неисправной вилки, неисправного шнура питания или неисправности самого прибора.

4. Поврежденная изоляция

Горячий и нейтральный провода имеют изоляцию для предотвращения соприкосновения двух проводов и возникновения короткого замыкания. Однако со временем эта изоляция может изнашиваться или повредиться.Будьте осторожны, чтобы случайно не проткнуть электрические провода, проделав отверстия для гвоздей и шурупов в стене. Также следите за признаками того, что вредители прогрызают изоляцию и проводку, регулярно проверяя рабочие места и ремонтируя любые трещины или дыры, которые вы видите по всему дому.

Как определить короткое замыкание

Короткое замыкание в электрической системе обычно довольно очевидно. Могут перегореть предохранители или прерыватель цепи может многократно срабатывать. Часто, когда происходят эти события, вы будете слышать громкий хлопок.Если вы часто сталкиваетесь с этими событиями, как можно скорее вызовите электрика, чтобы он проверил вашу систему.

Если вы считаете, что в вашей домашней электрической системе где-то произошло короткое замыкание, обратитесь к электрику, имеющему опыт устранения этих проблем. Чтобы получить дополнительную информацию, советы и рекомендации по электричеству в вашем доме или на работе, подпишитесь на наш блог!

Что такое короткое замыкание и чем оно опасно?

Электроэнергетические системы на промышленных предприятиях, а также в коммерческих и институциональных зданиях предназначены для безопасного и надежного обслуживания нагрузок.Короткое замыкание — опасное явление. Ниже я объясню определение короткого замыкания простыми словами, и вы точно поймете, что это такое.

Что такое короткое замыкание в электричестве?

Короткое замыкание — это перегрузка по току, вызванная незначительным повреждением импеданса между проводниками с разными потенциалами. Это случайно и может быть следствием неуклюжести или неисправности оборудования.

Короткое замыкание опасно, потому что, когда происходит короткое замыкание, сопротивление цепи падает почти на ноль Ом.Это быстро увеличивает ток и нагревает проводники или оборудование.

Что вызывает короткое замыкание?

Короткое замыкание обычно возникает из-за случайного контакта или износа изоляции. Короткое замыкание опаснее перегрузки, потому что повреждение происходит практически мгновенно. Примеры коротких замыканий включают случайное прикосновение двух или более проводов, прикосновение или падение инструментов через проводники под напряжением, или случайное соединение между проводниками под напряжением и землей.Такие замыкания на землю могут варьироваться от нескольких ампер до максимально доступного тока короткого замыкания.

Должны быть предусмотрены защитные устройства для ограничения и отключения токов короткого замыкания до того, как их тепловое (нагрев проводников, электрические дуги) и механическое (электродинамические силы) воздействия станут вредными и опасными. Защиту от короткого замыкания могут обеспечить предохранители и автоматические выключатели. Их отключающая способность и время размыкания цепи должны соответствовать защищаемой цепи.

Чем опасно короткое замыкание?

Когда происходит короткое замыкание, электрический ток может в сотни или тысячи раз превышать нормальный рабочий ток. КЗ высокого уровня могут достигать 30 кА или 200 кА. Это может иметь серьезные негативные последствия.

Короткое замыкание должно быть прервано как можно быстрее, чтобы свести к минимуму повреждения, которые могут включать:

  • Высокие магнитные силы, которые деформируют и деформируют шины и связанные с ними распорки, не подлежат ремонту.
  • Серьезное повреждение изоляции.
  • Проводники плавления или испарения.
  • Испарение металла, включая шины в электрооборудовании.
  • Ионизированные газы.
  • Дуговое возгорание.
  • Взрывы.

Короткое замыкание может вас убить?

Короткое замыкание может убить человека. При коротком замыкании ток идет в обход нормальной нагрузки. Токи короткого замыкания могут варьироваться от долей ампера до 200 000 ампер и более.

Как короткое замыкание может вызвать пожар?

Короткое замыкание обычно возникает, когда провод с низким сопротивлением проходит через потребляющее устройство. Большее количество электронов будет проходить по пути наименьшего сопротивления, а не через потребляющее устройство. Короткое замыкание обычно вызывает чрезмерный ток в кабелях, что приводит к перегреву и, возможно, к возгоранию.

Что используется в электрической цепи для предотвращения короткого замыкания?

Патронные предохранители, предохранители HRC, автоматические выключатели, автоматические выключатели в литом корпусе, воздушные выключатели, ручные пускатели двигателей могут использоваться для предотвращения короткого замыкания.Эти устройства быстро отключаются, как только обнаруживают короткое замыкание.

Продолжить чтение

Короткое замыкание — причины и последствия

Что такое короткое замыкание?

Каждый раз, когда в сети происходит сбой, при котором в одной или нескольких фазах протекает большой ток, считается, что произошло короткое замыкание цепь .

Короткое замыкание — это просто соединение с низким сопротивлением между двумя проводниками, подающими электроэнергию в любую цепь.Это приводит к чрезмерному протеканию тока в источнике питания через «короткое замыкание» и может даже привести к выходу источника питания из строя.

Если в цепи питания есть предохранитель, он сработает и перегорит, размыкая цепь и прекращая прохождение тока. MCB также используется для защиты от короткого замыкания.

Короткое замыкание может происходить в цепи постоянного или переменного тока (постоянного или переменного тока). Если это закороченная батарея, она очень быстро разряжается и нагревается из-за большого тока.

Посмотрите видео о коротком замыкании в линии электропередачи 110 кВ ниже.

Что такое ток короткого замыкания?

Когда происходит короткое замыкание, через цепь протекает сильный ток, называемый током короткого замыкания .

Это поясняется приведенным здесь рисунком. На рисунке показан однофазный генератор напряжения V и внутреннего импеданса Z и , который подает на нагрузку Z.

Однофазный генератор напряжения V и внутреннего сопротивления Zi питает нагрузку Z

.

В нормальных условиях ток в цепи ограничен сопротивлением нагрузки Z.

Однако, если клеммы нагрузки закорочены по какой-либо причине, полное сопротивление цепи уменьшается до очень низкого значения; в данном случае Z i . Поскольку Z i очень маленький, по цепи течет большой ток. Это называется ток короткого замыкания .

Короткое замыкание и перегрузка

Люди обычно путаются с перегрузкой и коротким замыканием, поскольку они оба вызывают проблемы в системе одинаковым образом. Следует различать короткое замыкание и перегрузку.

Когда происходит короткое замыкание, напряжение в точке повреждения снижается до нуля, и ток аномально высокой величины течет по сети к точке повреждения.

С другой стороны, перегрузка означает, что на систему были возложены нагрузки, превышающие расчетные. В таких условиях напряжение в точке перегрузки может быть низким, но не нулевым. Условия пониженного напряжения могут распространяться на некоторое расстояние за пределы точки перегрузки в остальную часть системы.

Токи в перегруженном оборудовании велики, но значительно ниже, чем в случае короткого замыкания.

Что вызывает короткое замыкание?

Короткое замыкание в системе питания является результатом каких-то ненормальных условий в системе. Это может быть вызвано внутренними или внешними эффектами.

  1. Внутренние эффекты вызваны поломкой оборудования или линий передачи из-за ухудшения изоляции в генераторе, трансформаторе и т. Д.Такие неприятности могут быть связаны со старением утеплителя. несоответствующий дизайн или неправильная установка.
  2. Внешние эффекты , вызывающие короткое замыкание, включают нарушение изоляции из-за скачков молнии. перегрузка оборудования, вызывающая чрезмерный нагрев: механическое повреждение со стороны населения и т. д.

Последствия короткого замыкания

Когда происходит короткое замыкание, ток в системе увеличивается до аномально высокого значения, а напряжение в системе снижается до низкого значения.

Сильный ток из-за короткого замыкания вызывает чрезмерный нагрев, который может привести к пожару или взрыву . Иногда короткое замыкание принимает форму дуги и приводит к значительному повреждению системы.

Например, дуга в линии передачи, которая не устранена быстро, вызовет ожог проводника, что приведет к его разрыву, что приведет к длительному прерыванию линии.

Низкое напряжение , возникшее в результате неисправности, оказывает очень вредное влияние на обслуживание, оказываемое энергосистемой.Если напряжение остается низким даже в течение нескольких секунд, двигатели потребителя могут быть отключены , а генераторы в энергосистеме могут стать нестабильными .

Из-за вышеупомянутых пагубных последствий короткого замыкания желательно и необходимо отключать неисправную секцию и восстанавливать нормальные значения напряжения и тока как можно быстрее.

Короткое замыкание может привести к очень высокой температуре с из-за большого рассеивания мощности в цепи.Эта высокая температура может быть использована в приложении. Дуговая сварка — типичный пример практического применения нагрева из-за короткого замыкания.

Источник питания для дуговой сварки может обеспечивать очень высокие токи, протекающие через сварочный стержень и свариваемые металлические детали. Точка контакта между стержнем и металлическими поверхностями нагревается до точки плавления, сплавляя часть стержня и обе поверхности в единое целое.

Как именно происходит «короткое замыкание»?

Если вы читали наши статьи, то знаете, что короткое замыкание обычно считается плохим явлением в вашей электрической системе.Вот почему мы делаем все возможное, чтобы ваша проводка, розетки и автоматические выключатели находились в отличном состоянии, чтобы предотвратить короткое замыкание, а также другие электрические проблемы. Когда вы читаете об электробезопасности или оцениваете потенциальные опасности в вашей собственной системе, может быть полезно узнать, что вызывает эти проблемы. На этой неделе мы рассмотрим короткие замыкания и способы их предотвращения.

В точности то, что написано на жестяной банке.

Короткие замыкания — это именно то, что следует из названия; когда электрический поток завершает свой путь на более короткое расстояние, чем предполагалось изначально.Это может произойти в системах электропроводки, где провода повреждены или даже просто ослаблены. Электричество всегда стремится двигаться по пути наименьшего сопротивления как можно быстрее. Провода с неисправным соединением создают прекрасную возможность для электрического тока уйти в обход. К сожалению, этот альтернативный путь может быть через легковоспламеняющиеся материалы или даже через вас! Короткое замыкание может вызвать электрические ожоги, пожар или поражение электрическим током. Даже если вы не пострадаете, ваши приборы и электрическая система могут серьезно пострадать из-за паразитного электрического тока.

The Pièce de Résistance

Наиболее опасным аспектом короткого замыкания является то, что оно вызывает резкое изменение электрического сопротивления. Думайте об этом, как о толкании двери с прислоненным к ней диваном. Этот диван вызывает большое сопротивление, из-за чего дверь открывается труднее. Теперь представьте, что кто-то внезапно уносит кушетку без вашего ведома, и вы толкаете с той же силой, что и при сильном сопротивлении. Вы собираетесь пролететь через эту дверь! То же самое происходит и в случае короткого замыкания: теперь, когда сопротивление внезапно и резко падает, электрический поток резко возрастает, беспрепятственно перенося большой объем тока по новейшему неожиданному пути.

Эффективные профилактические меры

К счастью, современные дома были построены так, чтобы снизить риск коротких замыканий. Вот некоторые ключевые элементы, которые должны быть в любом доме для максимальной защиты:

  • Автоматический выключатель. Эти выключатели в вашем подвале отключают цепь всякий раз, когда обнаруживают изменение электрического тока.
  • Защита GFCI. Более чувствительные, чем ваш автоматический выключатель, эти устройства обычно используются в розетках рядом с водой для защиты от замыканий на землю, определенного типа короткого замыкания.
  • Защита AFCI. Дуга — это еще один тип короткого замыкания, которое возникает, когда незакрепленные провода заставляют электричество перескакивать с провода на провод. Они могут создать большое количество тепла и риск возгорания, поэтому защита от AFCI является обязательной!
  • Регулярное электрическое обслуживание. Износ — наиболее частая причина коротких замыканий, поэтому техническое обслуживание является ключом к электробезопасности. Ежегодные плановые осмотры гарантируют, что ваша система не выйдет из строя.

Короткие замыкания могут быть опасной проблемой, но, к счастью, большинство из них можно полностью предотвратить! Позвоните нашим электрикам сегодня, чтобы проверить систему электропроводки в вашем доме. Ведь ваша безопасность — наш приоритет номер один!

Короткое замыкание

— обзор

10.2.4 Коэффициенты диффузии кислорода и катионов в окалине оксида алюминия

Короткие замыкания, рассматриваемые для диффузии внутри оксидов в диапазоне температур 800–1000 ° C, представляют собой границы зерен оксидов. Согласно Харрисону [71], можно выделить три режима диффузии.Режим A определяется Dt≫d, d — размер зерна; в этом случае объемное проникновение превосходит размер короткого замыкания, особенно границ зерен. Режим C утверждает, что проникновение является непрерывным на границах зерен (Dt≪δ, δ — ширина границы зерен). Наконец, режим B — это промежуточный режим, в котором в диффузии участвуют три процесса: объем, границы зерен и боковая диффузия δ≪Dt≪d2.

Маркерные эксперименты, проведенные с 18 O, привели к определению коэффициентов диффузии кислорода через границы зерен оксида алюминия и через объем оксида.Учитывая графики ln [ 18 O] = f (x), первая часть кривой соответствует кажущейся диффузии кислорода. Из трех режимов диффузии для исследования межзеренной диффузии обычно выбирают режим B [72,73]. В этом режиме δ≪Dt≪d2 с δ шириной границы зерна и d размером зерна оксида.

Кажущаяся диффузия кислорода соответствует диффузии в объеме и по границам зерен оксида.Он выражен в случае дислокации и объемной диффузии [74], но может быть использован в случае межзеренной границы и объемной диффузии [75,76], если предположить, что начало профиля распределения кислорода соответствует D приложение (Α режим) [72]:

10.1Dapp = 1-fDb + fDgb

Где f — доля сайтов, связанных с границами зерен; f может быть выражено как

10,2f = 3δϕ

δ — ширина границы зерен (обычно принимаемая равной 1 нм), а ϕ — средний размер кристаллитов оксида. D приложение определяется из решения второго закона Фика:

10,3Cxt-CsCo-Cs = erfx2Dappt

, где C s — концентрация кислорода-18 на поверхности оксидной окалины, C o — естественная концентрация кислорода-18 в оксидном слое (0,2 ат.% [77]), а t — время диффузии.

Эта модель, которая связывает начало профиля распределения с кажущимся коэффициентом диффузии, недавно обсуждалась Fielitz и др. .[78], которые предположили, что первая часть профиля диффузии кислорода в поликристаллическом муллите соответствует коэффициенту объемной диффузии в режиме B, а не кажущемуся коэффициенту диффузии.

Вторая часть кривой диффузии, ln [ 18 O] = f ( x ), позволяет определить коэффициент диффузии границ зерен кислорода. Применяя модель Уиппла – Леклера [79–81], D gb выражается как:

10.4Dgbδ = 0,6614Dbt − ∂lnC∂x6 / 5−5 / 3 − Dapp

, где ∂lnC∂x6 / 5 — наклон кривой ln [ 18 O] = f ( x 6 / 5 ). Комбинируя уравнения (10.1) и (10.4), можно получить следующее соотношение:

10,51 − fDb + 0,661fδ4Dbt − ∂lnC∂x6 / 5−5 / 3 − Dapp = 0

. Коэффициент объемной диффузии кислорода, D b . Используя D приложение и D b соотношение (10.1) позволяет расчет D gb .

В экспериментах по диффузии кислорода-18 определены коэффициенты диффузии кислорода. В диапазоне температур от 1050 до 1200 ° C результаты представлены в таблице 10.3 [82]. Сравнение с литературой затруднено, так как очень мало экспериментов было выполнено с термически выращенной окалиной оксида алюминия [82–86] (рис. 10.7), тогда как многочисленные работы касались определения коэффициентов диффузии в синтетическом оксиде алюминия [87–98].

Таблица 10.3. Коэффициенты диффузии кислорода определены в α-Al 2 O 3 , выращенном на сплаве Fe-25Cr-5Al при 1050, 1100 и 1200 ° C.

1050 ° C 1100 ° C 1200 ° C
D приложение O (см 2 ⋅ с ) 10 — 14 1,1 × 10 — 13 1,6 × 10 — 12
D b O (см 2 ⋅ s — 1 ) 5.7 × 10 — 19 8,3 × 10 — 17 1,9 × 10 — 16
D фунт O (см 2 ⋅3 — 1 1,6 × 10 — 11 2,2 × 10 — 11 3,2 × 10 — 10

10,7. Сравнение коэффициентов диффузии кислорода или хрома в термически выращенном оксиде алюминия.

Эксперименты по диффузии, проведенные как на синтетическом, так и на термически выращенном оксиде алюминия, показали, что диффузия алюминия преобладает над диффузией кислорода на границах зерен оксида алюминия, но значения, рассчитанные для синтетического оксида алюминия, в значительной степени отличаются от значений, определенных для термически выращенных чешуек.Химический состав (особенно уровень примесей или изменение концентрации легирующей примеси) и / или микроструктура (размер зерен, переходное образование оксида алюминия и т. Д.) Двух типов оксида алюминия может объяснить эти различия [99,100], поэтому очень сложно сравнивать эксперименты по диффузии. на синтетическом и термически выращенном глиноземе.

Хром используется в качестве изотопного маркера алюминия. Принимая во внимание, что и катионы хрома, и катионы алюминия диффундируют одинаково, результаты, полученные для коэффициентов диффузии хрома в окалине оксида алюминия, могут быть применены к коэффициентам диффузии катионов алюминия в той же шкале.В самом деле, легче использовать изотоп хрома в качестве диффузионного компонента, чем использовать алюминий, у которого нет природного изотопа; можно использовать только искусственный изотоп алюминия 26 Al, который радиоактивен и очень дорог. Этот радиоактивный индикатор использовался в очень немногих работах [90,101].

Короткие замыкания: почему они возникают?

Короткое замыкание может быть причиной того, что электронное устройство не работает должным образом. Это происходит, когда электроны выбирают непреднамеренный путь.Соединение отрицательной и положительной клемм аккумулятора вызовет короткое замыкание. Вот более подробный взгляд на то, что такое короткие замыкания:

Причины короткого замыкания
Основная причина короткого замыкания — неправильное соединение двух компонентов устройства. Например, резистор может стать несбалансированным, если одну сторону подключить к положительному, а не отрицательному источнику энергии.

Часто плохие соединения возникают из-за небрежной пайки, когда кусок припоя капает на печатную плату.Другой распространенной причиной короткого замыкания является падение металлического предмета на печатную плату, например, после разрезания ножек резистора. Удаление металла может быть затруднительным, что может вызвать многократное короткое замыкание, если он упадет под доску. Короткое замыкание также может быть вызвано повреждением компонентов, например контактов транзистора.

Самый опасный тип короткого замыкания — это когда оголенный провод соприкасается с другим оголенным проводом из-за изношенной изоляции. Такое подключение может вызвать поражение электрическим током и даже искру, которая может стать причиной возгорания.Когда электроны выбирают более короткий путь, это снижает сопротивление, что увеличивает нагрев. Чем больше нагревается проволока или металл, тем выше вероятность ожогов и пожаров. Короткое замыкание также может быть результатом слабых соединений, которые позволяют пересекать нулевые провода под напряжением. Иногда проволока может оголиться из-за грызунов, пережевывающих шнуры.

Как обнаружить короткое замыкание
Очевидные признаки короткого замыкания — это когда вы видите искры, пламя или дым после нажатия выключателя. Поскольку короткое замыкание вызывает срабатывание автоматического выключателя, выходом является перевернуть выключатель в противоположном направлении.

Эффективный способ определить короткое замыкание — использовать мультиметр с устройством проверки целостности цепи. Это устройство издает звуковой сигнал, когда обнаруживает соединение между двумя своими измерительными датчиками. Используя датчики, вы просто касаетесь тех мест, которые хотите проверить на наличие соединений, и прислушиваетесь к звуковым сигналам.

Международный союз компонентов

Allied Components International специализируется на разработке и производстве широкого спектра стандартных магнитных компонентов и модулей, таких как индукторы для микросхем, магнитные индукторы на заказ и трансформаторы на заказ.Мы стремимся предоставлять нашим клиентам продукцию высокого качества, обеспечивать своевременные поставки и предлагать конкурентоспособные цены.

Мы — растущее предприятие в магнитной промышленности с более чем 20-летним опытом.

Что такое короткое замыкание? И почему это опасно?

Короткое замыкание может произойти случайно и повредить ваш дом и бытовую технику. Вот некоторая информация, которая поможет вам понять, что такое короткое замыкание и почему оно опасно.

Что такое короткое замыкание?

Короткое замыкание происходит, когда часть провода, по которому проходит ток, соприкасается с другим проводом и дает электричеству путь с меньшим сопротивлением. Вкратце, короткое замыкание дает электрическому пути наименьшего сопротивления между двумя точками, что означает, что короткое замыкание будет производить больше тепла и приведет к ожогам и пожарам.

Что вызывает короткое замыкание?

Шорты могут быть результатом множества проблем.Обратите внимание на некоторые из наиболее распространенных причин короткого замыкания.

  • Неисправная изоляция: Старая или неисправная изоляция позволяет контактировать нейтралью и проводам под напряжением, что может вызвать короткое замыкание.
  • Ослабленные соединения: Электрические насадки могут ослабнуть, и иногда нейтраль и провода под напряжением могут соприкасаться, что приводит к короткому замыканию.
  • Домашние вредители: Домашние вредители, такие как мыши, крысы и белки, иногда могут грызть провода.Это также может привести к пересечению нейтрального и токоведущего проводов и вызвать короткое замыкание.
  • Приборы: В старых или сломанных приборах со временем могут возникать короткие замыкания. Поэтому рекомендуется регулярно проверять их у специалиста.

Чем опасны короткие замыкания?

Короткое замыкание — это ненормальное электрическое соединение, которое позволяет дополнительному электричеству проходить через ваши переключатели, приборы и розетки. Дополнительное тепло, генерируемое дополнительным электричеством, также может вызвать возгорание поврежденных проводов и достичь легковоспламеняющихся частей вашего дома.

Оголенные или оборванные провода, а также пережеванные или поврежденные шнуры также могут стать причиной короткого замыкания. Поэтому, если вы заметили оборванный или поврежденный шнур, немедленно отключите его.

Добавить комментарий

Ваш адрес email не будет опубликован. Обязательные поля помечены *